Genito-urinary Flashcards

1
Q

A 50-year-old man attends his GP feeling generally lethargic. The GP organizes blood tests
and these reveal renal impairment. A subsequent ultrasound examination shows bilateral
hydronephrosis without obvious cause. A CT scan of the abdomen then demonstrates that the
hydronephrosis is secondary to bilateral ureteric obstruction from abnormal retroperitoneal soft
tissue, intimately related to the aorta and IVC. Which of the following features on CT would
suggest that the soft tissue is more likely due to retroperitoneal fibrosis, rather than a malignant
cause? [B1 Q38]

A. Nodular contour to the soft tissue.
B. Contrast enhancement of the soft tissue.
C. More severe hydronephrosis in the kidneys.
D. Close application of the soft tissue to the adjacent vertebrae.
E. Soft-tissue extension above and below the level of the renal hila.

A

Close application of the soft tissue to the adjacent vertebrae.

Unfortunately attempts to consistently distinguish benign retroperitoneal fibrosis (RPF) from
malignancy are fraught with danger, but there are certain CT findings which are more
commonly seen in one or other of these conditions.

Malignancy tends to be larger and bulkier, displaying mass effect and displacing the aorta and IVC anteriorly from the spine and the ureters laterally. The purely fibrotic process of benign RPF tends to tether these structures to the adjacent vertebrae

Malignancy is more likely to extend cephalad to the renal hila, with benign RPF remaining
caudal to the hila. Neoplasia also more typically has a nodular outline, whereas benign RPF
usually manifests as a plaque-like density. There are, of course, exceptions to both these
features.

Contrast enhancement is not a reliable feature for distinguishing benign RPF from malignancy,
as both malignancy and active RPF can enhance with contrast. Similarly, the degree of
hydronephrosis caused is not a good distinguisher.*

How well did you know this?
1
Not at all
2
3
4
5
Perfectly
2
Q

Regarding retroperitoneal fibrosis (RPF): [B3 Q 42]

A. Is common in females in the primary form
B. Beta-blockers are a common cause
C. Desmoplastic response to malignancy is the most common case in secondary RPF
D. Causes lateral deviation of the mid ureter
E. In the primary form responds to steroids

A

E

Two thirds of cases of RPF are primary and one third are secondary. Both forms are more
common in males. Secondary causes include drugs such as methysergide, beta-blockers,
phenacetin. Medial deviation of the ureter occurs in the mid third.

How well did you know this?
1
Not at all
2
3
4
5
Perfectly
3
Q

A 60-year-old male, treated long term for hypertension with hydralazine, develops bilateral
hydronephrosis
. On CT KUB, the ureters are deviated medially and obstructed by a large,
plaque-like, para-aortic, soft-tissue density. The aorta appears ‘taped-down’ to the vertebral
column
rather than elevated by the para-aortic tissue. Which of the following is the most likely
diagnosis? [B4 Q14]

a. enlarged retroperitoneal lymph nodes due to Hodgkin’s disease
b. enlarged retroperitoneal lymph nodes due to non-Hodgkin’s lymphoma
c. retroperitoneal fibrosis
d. bilateral ureteral transitional cell carcinoma
e. metastatic lymph node enlargement from testicular embryonal cell carcinoma

A

Retroperitoneal fibrosis

Retroperitoneal fibrosis can cause extrinsic compression of both ureters and retroperitoneal
vascular structures such as the aorta, inferior vena cava and iliac vessels. It can be idiopathic
or secondary to inflammatory aortic aneurysm, retroperitoneal metastases, haemorrhage,
abscess, urinoma, diverticulitis, appendicitis, Crohn’s disease, and drugs such as ergot
alkaloids and hydralazine.

Malignant retroperitoneal lymphadenopathy causing ureteric obstruction tends to encircle
the aorta, elevating it off the vertebral column. In contrast, retroperitoneal fibrosis rarely
extends between the aorta and the vertebrae, and therefore appears to tape the aorta down
to the spine.

How well did you know this?
1
Not at all
2
3
4
5
Perfectly
4
Q

A 65-year-old man with known abdominal aortic aneurysm and under follow up for lymphoma,
presents with backache. Contrast-enhanced CT shows a doughnut shaped soft tissue mass
surrounding the lower part of aneurysmal abdominal aorta and the ureters are pulled medially
with bilateral hydronephrosis. What is the most likely diagnosis? [B5 Q21]

(a) Retroperitoneal fibrosis
(b) Lymphoma recurrence
(c) Aneurysm leak
(d) Radiation injury
(e) None of the above

A

Retroperitoneal fibrosis

This is hard fibrous tissue enveloping the retroperitoneum, including the great vessels, ureters
and the lymphatics. The plaque typically begins around the aortic bifurcation and extends
cephalad to the renal hilum, and it rarely extends below the pelvic brim.

How well did you know this?
1
Not at all
2
3
4
5
Perfectly
5
Q

A 28-year-old woman suffers blunt injury to her abdomen following a road traffic accident. A
polytrauma CT scan does not demonstrate any intra-abdominal injuries, but there are features indicating retroperitoneal injuries. Regarding these features, which of the following is true?
[B2 Q18]

a. Retroperitoneal air may indicate pulmonary injuries
b. Haematomas in the posterior pararenal space do not extend into the pelvis
c. The most common region demonstrating retroperitoneal haemorrhage following trauma is
usually around the aorto-caval region in the midline
d. Adrenal injuries are more common on the left
e. Low-attenuation fluid (<20HU) in the retroperitoneum is always indicative of injury to the
pelvi-calyceal system or the ureters

A

Retroperitoneal air may indicate pulmonary injuries

  • Air in the retroperitoneum can follow pneumothorax. However, in the absence of
    pneumothorax, it is strongly indicative of duodenal/colonic injury.
  • The posterior and anterior pararenal spaces communicate freely with the pelvic
    retroperitoneum, whilst the perinephric space is enclosed
    .
  • The retroperitoneum is divided into three zones: I – midline retroperitoneum; II – lateral
    retroperitoneum; and III – pelvic retroperitoneum. Zone III is the commonest site for
    haematoma following blunt injury.
  • Adrenal injuries are more common on the right.
  • Low-attenuation fluid can be seen even in the absence of urine leak, usually indicating
    hypoperfusion shock syndrome.
How well did you know this?
1
Not at all
2
3
4
5
Perfectly
6
Q

A 60-year-old female presents with a large abdominal mass. CT demonstrates a large
retroperitoneal fat-containing mass. Which of the following is true about the different fat-
containing retroperitoneal masses? [B2 Q28]

a. Predominantly low signal on T1-weighted and a high signal on T2-weighted images preclude
a diagnosis of liposarcoma
b. Calcification within a liposarcoma is usually associated with a better prognosis
c. Lipomas are rare in the retroperitoneum
d. An extremely FDG-avid retroperitoneal fat-containing tumour is almost certainly a
liposarcoma
e. Given time, most lipomas will dedifferentiate into liposarcomas

A

Lipomas are rare in the retroperitoneum

Liposarcomas are the most common sarcomas in the retroperitoneum. Whilst well differentiated liposarcomas are the commonest, myxomatous and dedifferentiated liposarcomas can have varying appearances and so low T1-weighted signal does not preclude a diagnosis of liposarcoma.

Lipomas, whilst exceedingly rare in the retroperitoneum, almost undergo malignant changes. Whilst liposarcomas can have minimal-to-increased FDG uptake, a very FDG-avid fat-containing retroperitoneal tumour is quite likely a hibernoma.

How well did you know this?
1
Not at all
2
3
4
5
Perfectly
7
Q

A 58-year-old smoker presents with haemoptysis and chest pain. A CT of chest confirms a lung
carcinoma. While reporting the CT you notice that there is enlargement of the right adrenal
gland. The patient has already left the department, but by chance is due to have an MRI scan
of lumbar spine at a nearby institution the following morning. Due to time constraints, they can
only fit in one sequence to image the adrenal glands. Which one sequence is of most use in
further characterizing the adrenal abnormalities? [B1 Q35]

A. T2WI.
B. STIR.
C. Axial T1WI with fat saturation.
D. In- and out-of-phase T1WI.
E. DWI.

A

In- and out-of-phase T1WI.

This is a T1WI technique and will reveal the presence of intracellular lipid (microscopic fat) via a dropout of
signal on the out-of-phase imaging when compared to the in-phase imaging. Thus, a benign
adrenal adenoma will show such signal dropout (20% or greater in quantity is diagnostic; 10–
20% is highly suggestive), whilst adrenal metastases will not.

However, approximately 15% of benign adenomas do not accumulate intracellular lipid and
may retain signal on the out-of-phase imaging; in such cases, dynamic gadolinium-enhanced
images can increase specificity to over 90–95% (by showing washout characteristics as seen
on CT).

If there is still doubt, PET-CT is useful. Adrenal hyperplasia may also show loss of signal on the
out-of-phase imaging. Of note an adrenal cortical carcinoma can show dropout of signal in
portions on the out-of-phase sequence, but there is not the uniform loss of signal as seen
with adenomas.

The T1WI with fat saturation will show signal dropout in areas of extracellular lipid, e.g.
macroscopic fat
in lipomas, dermoid cysts, or the subcutaneous tissues. T2WI is not of much
benefit in distinguishing benign from malignant adrenal masses unless the tumour is a
phaeochromocytoma, which can show very high T2WI signal.

How well did you know this?
1
Not at all
2
3
4
5
Perfectly
8
Q

A 64-year-old patient is referred for a CT of abdomen 10 days post laparotomy for a right
hemicolectomy for colonic adenocarcinoma. His post-operative course is initially uneventful,
but the request form states that over the last 2 days he has developed pyrexia and today his
inflammatory markers are markedly raised, he is ‘septic’ and unwell. The surgeons suspect a
perforation or anastomotic leak, but you find no significant free fluid or air. There is marked
bilateral enhancement of the adrenal glands, which are normal in size. The remainder of the
abdominal viscera are unremarkable and the IVC and aorta are normal in calibre. There is
marked consolidation at both lung bases. What is the significance of the appearance of the
adrenal glands? [B1 Q46]

A. Hypovolaemic shock.
B. Phaeochromocytoma.
C. Hypervascular metastases.
D. Addisonian crisis secondary to tuberculous adrenalitis.
E. Adrenal hyperplasia as a response to the recent surgery

A

Hypovolaemic shock.

Marked adrenal enhancement may be the only sign of significant hypovolaemic shock. This is
thought to be due to hyper-perfusion of the adrenal glands because of their crucial role in
this clinical situation. Other signs that may accompany this sign, the ‘hypoperfusion complex’
described in shock due to trauma, are collapsed IVC, small hypodense spleen, small aorta and
mesenteric arteries, shock nephogram (lack of renal contrast excretion), intense pancreatic
enhancement, dilatation of fluid-filled intestine with thickening of folds, and increased
enhancement of the wall. However, in cases of hypovolaemic shock due to sepsis, where
there has been rapid fluid replacement, the IVC and aorta may be of normal calibre
and
persisting marked adrenal enhancement has been described as the only abnormality.

Phaeochromocytomas are bilateral in only approximately 10% and this would be even less
likely in the presence of another neoplasm. Hyper-vascular metastases are uncommon in
colonic carcinoma. Adrenal hyperplasia may occur as a response to stress, but the adrenals
would enhance normally
. In tuberculous adrenalitis, the adrenal glands show areas of
necrosis and sometimes calcification, with possible rim enhancement. In all four of these
alternative options, the adrenal glands would be enlarged.

How well did you know this?
1
Not at all
2
3
4
5
Perfectly
9
Q

A 2 cm adrenal lesion with an attenuation value of 20 HU is seen on a non-contrast CT of a
patient with lung cancer. The following are all true except: [B2 Q40]

a. A 60% washout on delayed post-contrast CT would be in keeping with an adenoma
b. A signal intensity decrease of 40% or more on chemical shift imaging indicates malignancy
c. PET-CT is interpreted as positive if the FDG uptake of the adrenal lesion is greater than that
of the liver
d. Functioning adrenal adenomas can be a cause for false positives on PET-CT
e. PET-CT has somewhat higher and more consistent accuracy than dynamic CT or chemical
shift MR imaging.

A

A signal intensity decreases of 40% or more on chemical shift imaging indicates malignancy

A signal intensity decreases of less than 20% is usually indicative of malignancy in an adrenal
lesion.

How well did you know this?
1
Not at all
2
3
4
5
Perfectly
10
Q

A 54-year-old female patient presents with anaemia and haematuria. A CT of abdomen
confirms renal cell carcinoma of the right kidney, but there is also enlargement of the right
adrenal gland. Which of the following CT characteristics is most consistent with a benign
adrenal adenoma? [B1 Q41]

A. A pre-contrast attenuation of 50.
B. An immediate post-contrast attenuation of 50.
C. A relative percentage washout (RPW) of 60%.
D. Lesion size of 50 mm.
E. Heterogeneity of the lesion.

A

A relative percentage washout (RPW) of 60%.

  • Findings consistent with an adrenal adenoma are:
  1. a pre-contrast attenuation of 10 HU or less,
  2. an absolute percentage washout (APW) of 60% or greater, or
  3. an RPW of 40% or greater.

The percentage washout is calculated by comparing the attenuation value at 15 minutes post
contrast (delayed H), to the value in the portal venous phase (enhanced H), and in the case of
APW, the pre-contrast value.

RPW = 100 × (enhanced H – delayed H)/ (enhanced H)
APW = 100 × (enhanced H – delayed H)/ (enhanced H – pre-contrast H)

In practice, an unenhanced scan is not usually performed and thus only the RPW is calculated.

Adrenal cortical carcinomas usually have an RPW of less than 40% although exceptions have
been reported. Their large size (usually greater than 6 cm), heterogeneity pre-contrast
(necrosis), and heterogeneous enhancement are more reliable indicators of the diagnosis.
Phaeochromocytomas and hyper-vascular metastases may mimic adenomas, but most
metastases show RPW < 40% and APW < 60%.

How well did you know this?
1
Not at all
2
3
4
5
Perfectly
11
Q

A man is found to have a single adrenal mass of diameter 35 mm. On an unenhanced CT scan,
the average attenuation value is 30 HU. On a CT timed at 60 seconds after iodinated contrast
medium injection, the attenuation value of the mass is 90 HU. By 15 minutes after contrast, the
attenuation value is 50 HU. Which of the following is the most likely diagnosis? [B4 Q37]

a. lipid-rich adenoma
b. lipid-poor adenoma
c. metastasis
d. adrenal cortical cancer
e. adrenal haemorrhage

A

Lipid poor adenoma

An unenhanced CT attenuation value of less than 10 HU is in keeping with a lipid-rich
adenoma. With a threshold of 60% or higher for absolute contrast-enhancement washout, a
sensitivity of 98% and specificity of 92% can be achieved in differentiating adenomas from
non-adenomas. Percentage of enhancement washout ¼ ([attenuation at 60 s – attenuation
at 15min] / [attenuation at 60 s – attenuation on plain CT]) x 100. Applying this to the figures
quoted in the question gives an absolute washout of around 66.6%.

How well did you know this?
1
Not at all
2
3
4
5
Perfectly
12
Q

A 70-year-old, who is a lifelong smoker, is investigated for weight loss. Among other findings,
an adrenal nodule of 3 cm short axis diameter is found on post-contrast CT, with an average
attenuation value of 60 HU. On in-phase T1W images, the adrenal nodule is isointense to spleen;
on out-of-phase T1W images, the whole of the nodule returns significantly lower signal than
the spleen. Of the following, which is the most likely diagnosis for this adrenal nodule? [B4
Q75]

a. lung cancer metastasis
b. collision tumour
c. adrenal adenoma
d. phaeochromocytoma
e. hyperfunctioning adrenal cortical neoplasm

A

Adrenal adenoma

Signal dropout during out-of-phase T1W sequences occurs in lipid-rich adenomas by virtue of
their fat content. Adrenal primaries and metastases do not share this feature. Collision
tumours arise when a metastasis occurs in an adrenal gland that already contains an adenoma,
in which case signal characteristics of both are seen on the T1W in- and out-of-phase
sequences

How well did you know this?
1
Not at all
2
3
4
5
Perfectly
13
Q

A 52-year-old male smoker has been recently diagnosed with bronchogenic carcinoma with
cerebral metastasis. Staging CT shows a 1.5 cm nodule in the left adrenal gland. On MRI, the
nodule is isointense to spleen on T2 and shows marked hypo-intensity on out-of-phase GRE
images. What is the most likely diagnosis? [B5 Q16]

(a) Adrenal metastasis
(b) Adrenal adenoma
(c) Adrenocortical carcinoma
(d) Adrenocortical hyperplasia
(e) Adrenal cyst

A

Adrenal adenoma

This is the typical feature of adrenal adenoma and is seen in more than 95% adenomas. The
fat/lipid in the adenoma causes a chemical shift artefact which results in significant loss of
signal on out-of-phase GRE images.

How well did you know this?
1
Not at all
2
3
4
5
Perfectly
14
Q

A 45-year-old man presents with left-sided pain in abdomen. CT shows a 5 cm mass in the left
adrenal gland, predominantly containing tissues with Hounsfield units of approximately -80.
On MRI, the lesion high signal on T1 and low signal on STIR sequence. What is the most likely
diagnosis? [B5 Q32]

(a) Liposarcoma
(b) Adrenal myelolipoma
(c) Adrenal carcinoma
(d) Adrenal metastases
(e) Pheochromocytoma

A

Adrenal myelolipoma

Given the negative Hounsfield units on CT and loss of signal on fat suppression, the lesion
contains predominantly fat. These are benign tumours containing fat and haematopoietic
tissue. Presence of fat in an adrenal lesion is highly suggestive of a myelolipoma.

How well did you know this?
1
Not at all
2
3
4
5
Perfectly
15
Q

A 36-year-old male patient presents with abdominal pain. He has a history of hypertension and
obesity
. A CT of abdomen reveals a 6-cm right adrenal mass, which shows heterogenous but
peripheral enhancement, necrosis, and some calcification. There is early invasion of the IVC.
The left adrenal gland is atrophied. What is the most likely diagnosis? [B1 Q30]

A. Neuroblastoma.
B. Adrenal cortical carcinoma.
C. Myelolipoma.
D. Adrenal adenoma.
E. Phaeochromocytoma

A

Adrenal cortical carcinoma.

The clinical picture is one of undiagnosed Cushing’s syndrome with obesity and hypertension.
In this case it is adrenocorticotropic hormone (ACTH) independent Cushing’s, as the negative
feedback from the cortisol producing adrenal carcinoma causes reduction in ACTH levels and
atrophy of the contra-lateral, normal adrenal gland.

Adrenal adenoma can cause Cushing’s syndrome, but the features described point to adrenal
carcinoma. They have a bimodal distribution (first and fourth decades). On average 55% are
functional, manifesting with Cushing’ syndrome, feminization, virilisation, or a mixture of
these. Hypertension is common in all syndrome types.

Most masses measure more than 6 cm. They are heterogenous on unenhanced CT, owing to
necrosis. They enhance heterogeneously, often peripherally, with a thin rim of enhancing
capsule in some cases. In 19–33% of cases calcification or microcalcifications have been
identified. The liver is the most common metastatic site, followed by the lung and lymph
nodes. Direct extension and tumour thrombus can also occur. Compression of the IVC can
lead to presentation with abdominal pain, lower extremity oedema or pulmonary embolism.

Neuroblastoma is a disease of childhood. Myelolipoma is a relatively uncommon benign
adrenal mass containing fat and haemopoietic tissue.

Phaeochromocytoma is classically brightly enhancing but can have a variety of CT
appearances. It would explain hypertension, but not atrophy of the contra-lateral adrenal
gland
. Phaeochromocytoma rarely invades the IVC.

How well did you know this?
1
Not at all
2
3
4
5
Perfectly
16
Q

A 40-year-old female is found to have a suspected incidental left adrenal lesion on ultrasound.
Which of the following CT or MR features is least likely in a phaeochromocytoma? [B2 Q29]

a. High signal on T2-weighted images
b. Avid enhancement post-gadolinium injection
c. Mean lesion attenuation of more than 10HU
d. Less than 40% washout on delayed CT scanning
e. Calcification

A

Calcification

Whilst phaeochromocytomas can have varied appearances on CT and MR, typically they are
high on T2-weighted and low on T1-weighted images and enhance avidly post-contrast. They
normally have an attenuation value of more than 10 HU, but calcification is seen in only about
10% of cases.

How well did you know this?
1
Not at all
2
3
4
5
Perfectly
17
Q

A middle-aged woman presenting to the medical team with headaches, palpitations,
tachycardia, and hypertension is suspected to have a phaeochromocytoma. You are asked
advice on imaging modalities. Which one of the following statements is true regarding the
imaging characteristics of a phaeochromocytoma? [B2 Q36]

a. I-131 MIBG imaging is only 20% sensitive for phaeochromocytoma
b. Poor contrast enhancement on CT
c. Bilateral in 25% of cases
d. Usually hypovascular on angiography
e. No change in signal intensity between in-phase and out-of-phase T1-weighted MRI images

A

No change in signal intensity between in-phase and out-of-phase T1-weighted MRI images

There is no change between the in-phase and out-of-phase imaging on MRI as there is very
low-fat content in phaeochromocytoma. MR is the method of choice for imaging and usually
(60%) the phaeochromocytoma will be hyperintense to spleen on T2-weighted imaging.

Angiography can localise the lesion in >90% of cases. Appearance on ultrasound can be
variable with about 70% appearing as solid lesions whilst 15% are cystic. The ‘rule of tens’
applies to phaeochromocytoma, i.e., 10% are bilateral, 10% are extra-adrenal, 10% are
malignant and 10% are familial.

How well did you know this?
1
Not at all
2
3
4
5
Perfectly
18
Q

A 50-year-old man has surgery to remove a tumour confined to the adrenal gland. Histology
reveals a phaeochromocytoma. Subsequently, he develops hypertension and urinary
vanillylmandelic acid is found to be elevated. An MIBG scan is performed. Activity in which
of the following organs is most likely to be a metastasis? [B4 Q31]

a. lung
b. bladder
c. thyroid
d. colon
e. spleen

A

Lung

Normal MIBG uptake is seen in myocardium, liver, spleen, bladder, adrenal glands, salivary
glands, nasopharynx, thyroid and colon. Abnormal MIBG activity is seen in
phaeochromocytoma (paraganglioma when extra-adrenal), neuroblastoma, carcinoid
tumour, medullary thyroid carcinoma and ganglioneuroma. Ten per cent of
phaeochromocytomas are familial, 10% bilateral or multiple, 10% extra-adrenal and 10%
malignant. Metastatic spread is to bone, lymph nodes, liver, and lung.

How well did you know this?
1
Not at all
2
3
4
5
Perfectly
19
Q

A 57-year-old hypertensive woman presents with recurrent abdominal pain. Urine shows
elevated levels of vanillylmandelic acid. CT shows a large mass at the superior pole of right
kidney. On MRI, the lesion is heterogenous, and appears low signal on T1 and high signal on
T2 with enhancement with gadolinium. What is the most likely diagnosis? [B5 Q27]

(a) Lymphoma
(b) Renal cell carcinoma
(c) Pheochromocytoma
(d) Retroperitoneal liposarcoma
(e) Nodal metastasis

A

Pheochromocytoma

This tumour usually arises from the adrenal medulla. Note the 10% rule: 10% are extra-
adrenal, 10% malignant and 10% bilateral. MRI features are typical and with elevated urine
vanillylmandelic acid levels it is diagnostic.

How well did you know this?
1
Not at all
2
3
4
5
Perfectly
20
Q

In a 72-year-old man undergoing abdominal CT for ongoing lower abdominal pain, a 2 cm
right-sided adrenal lesion is detected. He has no history of malignant disease. Which of the
following parameters would be more in keeping with a malignant than a benign adrenal lesion?
[B5 Q24]

a. Size of 2.5cm
b. Hounsfield units of 8 on non-enhanced CT
c. Washout of >60% when comparing non-enhanced CT with contrast-enhanced CT
d. Loss of signal within the lesion on out-of-phase MRI imaging
e. Maximum standardised uptake value >4 on FDG-PET

A

Maximum standardised uptake value >4 on FDG-PET

This is suspicious for metastatic malignant disease with the most common primary sites being
lung, colon, melanoma, and lymphoma. An incidental adrenal lesion is detected on 1% of
abdominal CT. Even in the presence of a known malignancy, 87% of incidental lesions less
than 3 cm in size are benign. Other features suggestive of malignancy are large size,
irregularity, and inhomogeneity.

How well did you know this?
1
Not at all
2
3
4
5
Perfectly
21
Q

In dynamic renal imaging: [B3 Q24]

A. The patient must not eat or drink for 6 hours prior to the test
B. Provides information on total and divided function only
C. Total divided renal function are evaluated in addition to rates of transit through parenchyma
and outflow track
D. Diethylene triamine Penta acetic acid (DTPA) has the advantage of higher renal
concentration than inulin
E. DTPA is excreted by glomerular filtration and tubular excretion

A

Total divided renal function is evaluated in addition to rates of transit through parenchyma and outflow track

This investigation requires a hydrated patient to lie supine with knees slightly flexed to reduce
lumbar lordosis. DTPA is handled the same way as inulin. D and E are correct for MAG3

How well did you know this?
1
Not at all
2
3
4
5
Perfectly
22
Q

A 35-year-old female has investigations for episodic right loin pain. Ultrasound scan of the
renal tract is unremarkable. A DMSA scan is performed with the patient sitting and shows only
30% contribution to the total tracer activity from the right kidney. When the counts are repeated
supine, the contribution from the right kidney is 50%. What is the most likely abnormality of
the right kidney? [B4 Q26]

a. nutcracker kidney
b. nephroptosis
c. pelviureteric junction obstruction
d. ureteric calculus
e. vesicoureteric reflux

A

Nephroptosis

Ptosis of the mobile kidney when erect can cause symptoms and underestimation of
parenchymal DMSA uptake. Since the differential function may be a factor in considering
removal of a kidney, the technique should account for the possibility of nephroptosis
influencing the counts. A nutcracker kidney is a rare cause of left-sided loin pain and
haematuria; it is caused by compression of the left renal vein between the aorta and superior
mesenteric artery.

How well did you know this?
1
Not at all
2
3
4
5
Perfectly
23
Q

A 65-year-old male being investigated for microscopic haematuria has an ultrasound scan,
which suggests a 20 mm tumour in the cortex of the interpolar region of the left kidney. CT
scan confirms an enhancing mass in the same location. On DMSA SPECT, this abnormality
has good uptake. Which of the following is the most appropriate management? [B4 Q30]

a. no further action
b. biopsy
c. nephrectomy
d. image-guided drainage
e. chemotherapy

A

No further action

The abnormality described is prominent or hypertrophic cortex since it takes up DMSA, which
in the kidneys is a parenchymal tracer. Renal cell carcinoma, cysts, abscess, haematoma, scar
and infarct would be seen as photopenic areas on DMSA SPECT
, if large enough

How well did you know this?
1
Not at all
2
3
4
5
Perfectly
24
Q

A 73-year-old male diabetic patient, with poorly controlled hypertension, is referred for renal
Doppler ultrasound due to an episode of flash pulmonary oedema. He has a history of stage 3
chronic kidney disease. The ultrasound shows a small left kidney, which measures 5 cm in
bipolar diameter. The right kidney is also small, measuring 6 cm. The resistive indices measure
0.9 on both sides. The peak systolic velocity is 130 cm/s on the left and 150 cm/s on the right.
Which interventional treatment would be recommended for this patient? [B1 Q24]

A. Renal artery angioplasty on left side.
B. Renal artery stenting on left side.
C. Renal artery stenting on right side.
D. Bilateral renal artery stenting.
E. No intervention

A

No intervention.

The factors described are all indicators of poor outcome following renal artery intervention.
Reduced renal size bilaterally indicates advanced bilateral renal disease, unlikely to respond
to intervention. The renal ultrasound Doppler patterns are also not suggestive of renal artery
stenosis, which is indicated by a peak systolic velocity of greater than 180 cm/s. Resistive
indices of greater than 0.7 indicate a likelihood of improvement after intervention

How well did you know this?
1
Not at all
2
3
4
5
Perfectly
25
Q

You are the interventional radiology fellow in your hospital. A nephrologist has asked you for
an opinion on four patients they feel require renal arterial intervention.

Patient A is a 68-year-old male who has had a catheter angiogram which showed a 60%
narrowing in the right renal artery. This patient has refractory hypertension.

Patient B is a 42-year-old female, also with refractory hypertension. She has had an MR
angiogram, which has shown several stenoses, with intervening mild aneurysm formation, in
the proximal right renal artery.

Patient C is a 72-year-old female with impaired renal function, who underwent captopril renal
scintigraphy. On the baseline study, there was a similar appearance of the kidneys. On the
captopril study, there was a differential split of renal function of 15% between the right and
left kidney, with a decrease of time to peak activity of 320 seconds.

The final patient D had a renal Doppler ultrasound, which showed a biphasic flow pattern in
the segmental arteries, with a slow upstroke. Which of these patients has significant
atherosclerotic renal artery stenosis? [B1 Q29]

A. Patients A and B.
B. Patients C and D.
C. Patients A, C, and D.
D. Patients B, C, and D.
E. All of them.

A

Patients C and D

A 70% stenosis is taken as the cut-off for significant stenosis. Patient B is atypical in their age.
They also have imaging characteristics typical of fibromuscular dysplasia. Patients C and D
have classical imaging features of renal artery stenosis (RAS). Regarding ultrasound diagnosis
using Doppler, this is made by either showing flow acceleration immediately distal to the site
of stenosis or showing dampened flow in the segmental arteries. In the renal artery, a peak
systolic velocity of over 180 cm/s combined with a renal/aortic velocity ratio of over 3 is
reported as being the most sensitive method of detecting RAS.

How well did you know this?
1
Not at all
2
3
4
5
Perfectly
26
Q

You are consulted about a 24-year-old male patient who is day 3 following a blind right-sided
renal biopsy, which diagnosed glomerulonephritis. He has been well until today when he
developed acute right flank pain and haematuria. The urologists asked for an IVU prior to
assessing the patient. They feel this showed an absent kidney, but on the 15- and 20-minute
images, you feel you can detect a faint nephrogram that is slightly more pronounced on the 20-
minute image. Given your suspicions about the cause, which of the following would be the
least likely to contribute to the diagnosis? [B1 Q34]

A. Ultrasound.
B. MRI with GE sequences of the right kidney.
C. MRI with SE sequences of the right kidney.
D. Arterial phase CT abdomen.
E. Portal venous phase CT abdomen.

A

Arterial phase CT abdomen.

The IVU has surprisingly been helpful in showing a delayed increasing nephrogram, a classical,
although uncommon, appearance of renal vein thrombosis. This is also indicated by the
history of glomerulonephritis, which can manifest as nephrotic syndrome, the most common
cause for renal vein thrombosis.

Ultrasound is often successful at identifying a thrombosed right renal vein but is not useful in
assessing the longer left renal vein.

Gradient and SE sequences are both helpful in assessing the renal venous system, with GE
giving low-signal thrombus and SE giving high-signal thrombus.

Arterial phase CT would be too early to show a venous filling defect and thus may not
contribute, although it may show a rim nephrogram.

Portal venous phase CT is reasonably sensitive at detecting the lack of enhancement within
the renal vein and abnormal enhancement pattern within the affected kidney.

How well did you know this?
1
Not at all
2
3
4
5
Perfectly
27
Q

A 45-year-old male is diagnosed with renal cell carcinoma and is being worked up for curative
nephrectomy. Which one of the following imaging modalities would you advise as being the
most accurate at ruling out malignant renal vein invasion? [B2 Q19]

a. Doppler ultrasound
b. B-mode ultrasound
c. CT
d. MRI
e. PET-CT

A

MRI

MRI is superior to the other imaging modalities listed at ruling out renal vein invasion. CT is
still very accurate (reported as high as 96%), but MR has the advantage of being able to
accurately differentiate benign from malignant thrombus. MR offers no advantage in
detecting nodal disease, however, and patients being considered for curative surgery should
undergo staging CT of the chest, abdomen and pelvis. PET does not have a specific role for
detecting renal vein invasion.

How well did you know this?
1
Not at all
2
3
4
5
Perfectly
28
Q

A 32-year-old patient with a history of renal failure has undergone a renal transplant in your
centre. You are carrying out serial ultrasound investigations over the weekend to assess the
success of the transplant. The graft was a cadaveric transplant. The initial ultrasound scan
showed no evidence of peri-graft collection but did show mild renal pelvic dilatation. The
pulsatility index (PI) was 2.0 and the resistive index was 0.9. The patient was oliguric. Due to
the fears of acute rejection, the patient was commenced on cyclosporine and steroids. The
patient, now day 4, is currently anuric and has developed pain over the graft site. You have
done an ultrasound. The arterial flow in the kidney has changed. Proximally you notice reversal
of diastolic flow evident on spectral waveforms, but in other areas there is no flow evident in
the interlobar arteries. Flow is still present in the main renal artery. The RI is now 0.95 and the
PI is unchanged. There is still mild renal pelvic dilatation, but you notice a tubular structure of
intermediate echogenicity in the renal hilum. There is no evidence of peri-graft collection.
What do you think the most appropriate next investigative/ therapeutic step is? [B3 Q73]

A. Continue immunosuppression and further observation, with appropriate fluid administration.
B. Continue immunosuppression and renal biopsy.
C. Refer for immediate arterial intervention.
D. Refer for immediate venous intervention.
E. Nephrostomy.

A

Refer for immediate venous intervention.

There are several complications to be aware of in the immediate post-transplant period. The
most efficient and cost-effective method of surveillance for these is ultrasound.

Acute rejection and acute tubular necrosis (ATN) both occur early in the post-transplant phase
and can be difficult to differentiate. Both can cause elevation of pulsatility index (PI) (normal
<1.5) and resistive index (RI) (normal <0.7), although these tend to be higher with rejection.

ATN is more common with cadaveric transplants. It usually resolves within a matter of days
or weeks, but ultimately biopsy can be necessary to reliably differentiate these processes.
This patient probably had a degree of ATN initially.

Arterial occlusion is uncommon and is evident by reduced/absent arterial flow in the main
artery as well as in the interlobar arteries, which can demonstrate loss of flow with any cause
of severe oedema (e.g. renal vein thrombosis or acute rejection).

Renal vein thrombosis causes pain and anuria. It is associated with early treatment with
cyclosporine and steroids. It also causes a rise in the RI and PI. The key feature to notice is the
persistent flow in the main renal artery, reducing the likelihood of renal arterial thrombosis.
There is also a tubular structure noted in the hilum, which is what a thrombosed renal vein
looks like, and reversal of diastolic flow on arterial traces.

How well did you know this?
1
Not at all
2
3
4
5
Perfectly
29
Q

You are asked to assess a renal transplant kidney in the early post-operative period for any
abnormality and to serve as a baseline study. Which of the following findings on colour
Doppler ultrasound regarding the PI, RI, and B mode ultrasound imaging, regarding renal
morphology, would be considered within normal limits? [B1 Q74]

A. PI < 1.5, RI < 0.7, loss of cortico-medullary differentiation.
B. PI < 1.5, RI < 0.7, prominence of renal pyramids.
C. PI > 1.8, RI < 0.7, loss of cortico-medullary differentiation.
D. PI > 1.8, RI < 0.7, prominence of renal pyramids.
E. PI < 1.5, RI > 0.9, loss of cortico-medullary differentiation.
F. PI < 1.5, RI > 0.9, prominence of renal pyramids.
G. PI > 1.8, RI > 0.9, loss of cortico-medullary differentiation.
H. PI > 1.8, RI > 0.9, prominence of renal pyramids.

A

PI < 1.5, RI < 0.7, prominence of renal pyramids.

PI < 1.5 or RI < 0.7 can be regarded as normal, whilst a PI > 1.8 or RI = 0.9 should be regarded
as abnormal. Although both ATN and acute rejection cause the PI and RI to rise, the likelihood
of acute rejection is greater with higher values. Complete absence of diastolic flow, or flow
reversal, is due to acute rejection in most of the case.

Morphologically, the transplant kidney is very similar to the native and many of the subtle
differences are attributed to the improved resolution of the former. There is a well-defined
renal parenchyma peripherally, with a highly reflective echogenic sinus centrally. In
distinction to the native kidney, the renal pyramids are more commonly visualized within the
transplant, being hypoechoic, relative to the parenchyma itself.

Loss of cortico-medullary differentiation is pathological and is a B mode finding that can be
associated with acute rejection.

How well did you know this?
1
Not at all
2
3
4
5
Perfectly
30
Q

Which of the following causes immediate faint persistent nephrogram on Intravenous Urogram
(IVU)? [B3 Q13]

A. Renal vein thrombosis (RVT)
B. Acute obstruction
C. Acute hypotension
D. Acute tubular necrosis
E. Chronic obstruction

A

Renal Vein Thrombosis

Immediate faint persistent nephrogram
RVT
Acute glomerulonephritis
Chronic severe ischaemia

How well did you know this?
1
Not at all
2
3
4
5
Perfectly
31
Q

Which is the cause of rim nephrogram? [B3 Q14]

A. Acute complete arterial occlusion
B. Acute ureteric obstruction
C. Polycystic kidney disease
D. Medullary sponge kidney
E. Acute pyelonephritis

A

Acute complete arterial occlusion

Rim nephrogram
Acute complete arterial occlusion
Severe hydronephrosis

Striated nephrogram
Acute ureteric obstruction
Acute pyelonephritis
Acute renal vein thrombosis
AR polycystic kidney
Medullary sponge kindey

How well did you know this?
1
Not at all
2
3
4
5
Perfectly
32
Q

A 30-year-old man with known Hep B surface Antigen (HBsAg), abdominal pain, malaise and
weight loss is diagnosed with Polyarteritis nodosa (PAN). Which of the following is true? [B3
Q25]

A. Affects small arteries only
B. Causes aneurysms which are usually 5-10 mm in size
C. Produces moth-eaten nephrograms after thrombosis of the microaneurysms
D. Angiography has significantly higher sensitivity than specificity in diagnosis
E. Angiography has a much higher positive predictive value (PPV) compared with negative
predictive value (NPV)

A

Produces moth-eaten nephrograms after thrombosis of the microaneurysms

PAN
Affects medium and small arteries. Multiple microaneurysms 2-3 cm in size, which when thrombosed produce moth-eaten nephrogram. Angiography has high sensitivity and specificity but a much higher negative than positive predictive value in diagnosis. Diagnosis is often made with
clinical features and angiography.

How well did you know this?
1
Not at all
2
3
4
5
Perfectly
33
Q

A 29-year-old female with new onset hypertension has normal renal tract ultrasound. Magnetic
resonance Angiography (MRA) is performed which demonstrates narrowing at the distal right
main renal artery. Which is the most likely diagnosis? [B3 Q26]

A. Renal artery stenosis
B. Fibromuscular dysplasia
C. Arteriosclerotic RA disease
D. Neurofibromatosis
E. Polyarteritis nodosa

A

Fibromuscular dysplasia

FMD accounts for 35% of RAS. It is more common in children and young adults, affecting females > males. It is associated with hypertension and progressive renal insufficiency. FMD occurs bilaterally in 2 ⁄ 3 of cases and R > L 4:1. It usually affects the mid distal renal artery, and there can be a beaded appearance of artery.

How well did you know this?
1
Not at all
2
3
4
5
Perfectly
34
Q

A 55-year-old, hepatitis B-positive male under investigation for painless haematuria is admitted as an emergency with unilateral loin pain and hypotension. A renal arteriogram shows multiple, bilateral, small, renal artery branch aneurysms. Which of the following antibody titres is most likely to be positive? [B4 Q18]

a. anti-double-stranded DNA
b. anti-basement membrane
c. anti-Ro
d. anti-immunoglobulin G
e. perinuclear anti-neutrophil cytoplasmic

A

Perinuclear anti-neutrophil cytoplasmic - pANCA

While several forms of arteritis can cause multiple small renal aneurysms, it is polyarteritis nodosa that does so most commonly, affecting men more than women with a mean age of 55 years, the range being 18–81 years. It is associated with HIV and hepatitis B infection, and pANCA is usually positive.

Systemic lupus erythematosus: anti-double-stranded DNA antibodies
Goodpasture’s disease: anti-basement membrane antibodies
Sjogren’s syndrome: anti-Ro and anti-La antibodies
rheumatoid arthritis: anti-immunoglobulin G (anti-IgG), also known as rheumatoid factor

How well did you know this?
1
Not at all
2
3
4
5
Perfectly
35
Q

A 50-year-old male presents with history of weight loss, hypertension, and headaches. Bloods
show leucocytosis, eosinophilia and raised ESR. A selective renal angiogram shows bilateral,
multiple small vessel aneurysms. The renal arteries are normal. What is the most likely
diagnosis? [B5 Q17]

(a) Rheumatoid disease
(b) Polyarteritis nodosa
(c) Systemic lupus erythematosus
(d) Intravenous drug abuse
(e) Atrial myxoma

A

Polyarteritis nodosa

Appearances are virtually diagnostic of polyarteritis nodosa. The condition is characterised by
focal areas of necrotising arteritis with fibrinoid necrosis and small aneurysm formation in
skin, kidneys, cardiovascular system, and central nervous system

How well did you know this?
1
Not at all
2
3
4
5
Perfectly
36
Q

Which of the following causes immediate faint persistent nephrogram on Intravenous Urogram
(IVU)? [B3 Q13]

A. Renal vein thrombosis (RVT)
B. Acute obstruction
C. Acute hypotension
D. Acute tubular necrosis
E. Chronic obstruction

A

Acute glomerulonephritis, RVT and chronic severe ischaemia are all causes of immediate faint
persistent nephrogram.

How well did you know this?
1
Not at all
2
3
4
5
Perfectly
37
Q

A 54-year-old woman with no history of major illness is incidentally discovered to have a small,
solid enhancing lesion on CT at the lower pole of the right kidney. The CT has been performed
pre and post intravenous contrast. The lesion measures 9 mm in size. What is the most
appropriate management for this lesion? [B1 Q54]

A. Nephron sparing surgery.
B. Percutaneous biopsy.
C. Repeat CT in 3–6 months.
D. Right nephrectomy.
E. Lesion ablation.

A

Repeat CT in 3–6 months.

Solid masses smaller than 1 cm are challenging. Firstly, there is a reasonable chance that a
very small solid mass is benign
. Secondly, it is often difficult to characterize a mass smaller
than 1 cm as solid and enhancing
, despite a meticulous technique using state-of-the-art CT
and MR imaging. Thirdly, these masses are often too small to biopsy, therefore when
encountering a mass that is believed to be solid and is less than 1 cm in size, it is reasonable
to observe them with an initial examination with CT or MR at 3–6 months followed by yearly
examinations. A full work-up could ensue when the mass reaches 1 cm in size.

How well did you know this?
1
Not at all
2
3
4
5
Perfectly
38
Q

A 28-year-old male with a history of von Hippel Lindau disease and a 3-cm renal cell
carcinoma undergoes cryo-ablation. Which of the following is suggestive of incomplete
treatment/recurrence? [B1 Q62]

A. Hypodense ablation zone larger than the original tumour.
B. Lack of enhancement in the ablation zone.
C. Ablation zone unchanged in size over time.
D. Peripheral ring enhancement.
E. Perinephric fat-stranding post procedure.

A

Ablation zone unchanged in size over time.

Following ablation, the treatment zone is larger than the original lesion, as a margin of normal
tissue is intentionally ablated to prevent recurrence. This should reduce in size/involute over
time. If the ablation zone remains the same or increases in size, recurrence should be
suspected.

Lack of enhancement is a reliable indicator of successful treatment. It is not uncommon to
find peripheral enhancement in the immediate post-treatment period due to reactive
hyperaemia. Any nodular or central enhancement indicates incomplete treatment or
recurrence.

How well did you know this?
1
Not at all
2
3
4
5
Perfectly
39
Q

A 55-year-old man begins to feel unwell after RFA to his right kidney. Considering post-RFA
syndrome, which is the single best answer? [B3 Q1]

A. Flu-like symptoms and myalgia 10 days post procedure
B. Haemorrhage is the most common major complication
C. Urinomas from urine leaks are common
D. Cell death occurs at temperatures higher than 40’C with complete tumour necrosis at 50’ -
60’C
E. Involves conversion of high-pressure argon gas to cold low-pressure liquid by using the
Joule-Thomson effect

A

Haemorrhage is the most common major complication

Post-RFA syndrome (Flu-like symptoms) occurs 24-48 hours post-ablation. It is a likely inflammatory response to tumour necrosis or cytokine production. Persistence of fever beyond day 10 should be evaluated for infection. Haemorrhage is most seen in central tumours. Ureteral strictures and urinomas are rare.

How well did you know this?
1
Not at all
2
3
4
5
Perfectly
40
Q

Which of the following indicates T2 disease in renal cell carcinoma? [B3 Q28]

A. Perinephric fat involvement
B. Tumour size > 2.5cm
C. Tumour involvement of renal vein
D. Tumour involvement of adrenal gland
E. IVC involvement

A

Tumour size > 2.5 cm

Stage T2 disease is indicated when the tumour size is greater than 2.5cm.

How well did you know this?
1
Not at all
2
3
4
5
Perfectly
41
Q

Which of the following most favours RCC rather than TCC? [B3 Q31]

A. A central hypoechoic lesion on US
B. Intraluminal soft tissue mass in the calyx
C. Renal vein invasion
D. Infiltration of renal sinus
E. Contrast outlining tumour in the pelvis

A

Renal Vein Invasion

In TCC inferior vena cava and renal vein invasion are uncommon

How well did you know this?
1
Not at all
2
3
4
5
Perfectly
42
Q

A portal venous-phase CT of the abdomen and pelvis is performed in a 60-year-old man to
investigate upper abdominal and back pain, which is attributed to features of pancreatitis on
the scan. An incidental finding is of a rounded, renal lesion of diameter 3 cm, with average
attenuation value of 80 HU and containing no significant component with a negative
attenuation value on pixel densitometry. There are no previous images for comparison. What
is the most likely diagnosis of the renal lesion? [B4 Q2]

a. angiomyolipoma
b. renal cell carcinoma
c. simple cyst
d. high-density cyst
e. infected cyst

A

Renal Cell Carcinoma

A single portal venous phase CT is not the optimum image set to characterize renal
parenchymal lesions. However, renal cell carcinoma is more commonly encountered than
high-density cysts. Furthermore, carcinoma is most frequently found in men (2:1) aged over
50 years. Kidney neoplasms tend to have densities above 30 HU on an unenhanced CT and
rise by more than 10–20 HU post-contrast, usually being above 70 HU in the portal phase

How well did you know this?
1
Not at all
2
3
4
5
Perfectly
43
Q

CT scan of the chest, abdomen and pelvis is performed to stage a renal cell carcinoma. The
tumour arises in, and is confined to, the upper pole of the left kidney with a maximum
dimension of 5 cm. There is tumour thrombus in the left renal vein, inferior vena cava and right
atrium. There are no enlarged lymph nodes and no metastases seen. According to the TNM
classification what is the stage of the tumour? [B4 Q9]

a. T4 N0 M0
b. T2 N0 M0
c. T3a N0 M0
d. T3c N0 Mx
e. T3c N0 M0

A

T3c N0 M0

T1 <7 cm - Limited to kidney
T2 >7cm - Limited to kidney

T3 tumour extends beyond the kidney, into either the adrenal gland or
perinephric tissues (T3a), the renal vein or vena cava below the diaphragm (T3b) or the vena
cava above the diaphragm, or it invades the wall of the vena cava (T3c).

T4 tumour invades beyond Gerota’s fascia.

N1 or N2 nodal disease refers to involvement of a single regional node, or more than one regional node, respectively. Overall, T3c N0 M0 disease represents stage III disease

How well did you know this?
1
Not at all
2
3
4
5
Perfectly
44
Q

An 80-year-old male with a history of nephrectomy for renal cell carcinoma is found at follow-
up to have a heterogeneously enhancing 25 mm lesion confined to his remaining kidney. No
enlarged nodes or metastases are present. The lesion is biopsied and found to be an
adenocarcinoma. The patient decides upon radiofrequency ablation as treatment. A CT scan 1
month after the ablation quantifies the average post-contrast enhancement of the tumour as 8
HU. Which of the following best represents the degree of success of the radiofrequency
ablation? [B4 Q27]

a. failed
b. residual enhancing tumour requiring repeat ablation
c. residual enhancing tumour but no value in repeat ablation
d. successful
e. indeterminate

A

Successful

The practice of radiofrequency ablation of renal tumours is emerging. Currently, CT 1 month
after the procedure is used to assess treatment success. If enhancing prior to ablation, the
tumour is regarded as ablated if there is <10 HU rise in attenuation following contrast
administration. Bulky persistent irregular peripheral enhancement is the commonest
appearance of an incompletely treated lesion

How well did you know this?
1
Not at all
2
3
4
5
Perfectly
45
Q

A 35-year-old male with autosomal dominant polycystic kidney disease has been shown on CT
to have, among the innumerable renal cysts, several high-density cysts. Which of the following
MRI sequences would be most useful in detecting a renal cell carcinoma among haemorrhagic
or proteinaceous cysts? [B4 Q34]

a. T1W
b. T2W
c. T1W post-gadolinium
d. T1W fat-suppressed post-gadolinium
e. T1W post-gadolinium with pre-contrast T1W signal subtracted

A

T1W post-gadolinium with pre-contrast T1W signal subtracted

The cornerstone of diagnosis here is the post-contrast enhancement of renal cell carcinoma.
To identify this among the high T1 signal of haemorrhage or protein within cysts, it is ideal to subtract the precontrast T1 signal
. Fat suppression will not remove the distracting high signal from mildly complicated cysts. Risk of renal cell carcinoma is increased in adult polycystic renal disease when in renal failure. Hence, caution may be required since at certain levels of renal impairment the use of MRI contrast is not advised.

How well did you know this?
1
Not at all
2
3
4
5
Perfectly
46
Q

A 60-year-old man has a 4 cm rounded mass arising within the right kidney. It has
heterogeneous, strong post-contrast enhancement. Calcification is also evident within the
tumour. Which of the following features of this renal mass would favour the diagnosis of renal
cell carcinoma over angiomyolipoma? [B4 Q84]

a. marked vascularity
b. calcification
c. fat within the tumour
d. round morphology
e. hyperechogenic on ultrasound scan

A

Calcification

No calcification in AML whereas it is seen in 10% of renal cell carcinomas. Both these tumours can be hyper vascular. The cornerstone of diagnosis of an angiomyolipoma is identifying fat on CT or MRI; however, fat has been reported in renal cell carcinoma, and peripheral or renal sinus fat can become trapped in any large renal tumour

How well did you know this?
1
Not at all
2
3
4
5
Perfectly
47
Q

An adult male is initially investigated for abnormal liver function tests. Eventually, the
diagnosis of Stauffer’s syndrome is pronounced. What are the likely CT findings? [B4 Q96]

a. liver mass in keeping with hepatocellular carcinoma with renal metastases
b. renal mass in keeping with renal cell carcinoma with liver metastases
c. renal mass in keeping with renal cell carcinoma and hepatosplenomegaly without focal
hepatic or splenic lesions
d. hepatosplenomegaly and bilateral renal enlargement without focal lesions in any of these
organs
e. renal mass in keeping with renal cell carcinoma with a pancreatic head metastasis

A

Renal mass in keeping with renal cell carcinoma and hepatosplenomegaly without focal
hepatic or splenic lesions
.

Stauffer described a syndrome of nephrogenic hepatopathy in which a renal cancer without
liver metastases causes hepatosplenomegaly and abnormal liver function. Renal cell
carcinoma paraneoplastic phenomena include erythrocytosis and hypercalcaemia

How well did you know this?
1
Not at all
2
3
4
5
Perfectly
48
Q

A 67-year-old male patient presents with an 8-week history of left loin pain. A renal CT is
obtained, and this shows a 6-cm enhancing left renal lesion that has a fibrotic central scar. What
is the most likely diagnosis? [B1 Q14]

A. Renal leiomyoma.
B. Renal oncocytoma.
C. Renal metanephic adenoma.
D. Renal haemangioma (giant).
E. Renal juxta-glomerular cell neoplasm

A

Renal oncocytoma.

This is a benign renal cell neoplasm responsible for about 5% of all adult primary renal
epithelial neoplasms. It typically occurs in elderly men. They usually appear as solitary, well-
demarcated
, unencapsulated, fairly homogeneous renal cortical tumours with central stellate scar.

Bilateral, multicentric oncocytomas are seen in hereditary syndromes of renal oncocytosis and Birt–Hogg–Dubé syndrome. A central stellate scar is seen in approximately one-third. However, distinguishing them from RCC on imaging is not reliable.

Leiomyoma of the kidney is a benign smooth muscle neoplasm. It appears as a well-
circumscribed, homogeneous, exophytic solid mass that shows uniform enhancement on
contrast-enhanced CT. It may occasionally be cystic.

Metanephric adenoma is a benign renal neoplasm that is more common in middle-aged to
elderly females. It is associated with polycythaemia in 10%. It typically appears as a well-
defined, unencapsulated, solitary mass that may be hyperattenuating on unenhanced CT.
Calcification can be seen in up to 20%.

Hemangioma of the kidney occurs as an unencapsulated, solitary lesion that frequently arises
from the renal pyramids or the pelvis. Contrast-enhanced CT or MRI may show early intense
enhancement, with persistent enhancement on delayed images.

Juxtaglomerular cell (JGC) neoplasm or reninoma is an extremely rare, benign renal neoplasm
of myoendocrine cell origin, which is associated with a clinical triad of hypertension,
hypokalaemia, and high plasma renin activity. It typically appears as a unilateral, well-
circumscribed, cortical tumour and often measures less than 3 cm, but otherwise is
indistinguishable from other cortical neoplasms.

How well did you know this?
1
Not at all
2
3
4
5
Perfectly
49
Q

A 65-year-old male has a renal ultrasound scan for right flank pain which demonstrates a 7 cm
solid mass within the right kidney with a hypoechoic centre. Subsequent CT scan of the chest,
abdomen and pelvis reveals the lesion to have a low-attenuation central scar. There is no renal
vein invasion or evidence of malignancy elsewhere in the body. Which of the following is the
most likely diagnosis? [B2 Q9]

a. Lymphoma of the kidney
b. Transitional cell carcinoma
c. Collecting duct tumour
d. Oncocytoma
e. Nephroblastoma

A

Oncocytoma

Benign. Oncocytoma is a tubular adenoma that is very rarely malignant. They are often asymptomatic even when large. The central scar is typical and is due to haemorrhage and infarction of the tumour having outgrown its vascular supply.

Radiological differentiation from renal cell carcinoma can be very difficult and percutaneous needle biopsy is unreliable. Nephrectomy is therefore often indicated

How well did you know this?
1
Not at all
2
3
4
5
Perfectly
50
Q

Oncocytosis is considered in a 50-year-old man with bilateral renal masses. Which is the single
best answer regarding oncocytosis? [B3 Q3]

A. Account for 2% of all renal cortical tumours
B. Multifocality bilateralism and metachronous tumours occur together in 4-6%
C. In multifocal cases co-existent RCC is present in 50% of cases
D. Often have preserved renal function
E. The final diagnosis can be made on MR above

A

Multifocality bilateralism and metachronous tumours occur together in 4-6%

Co-existent RCC in 10% of cases. Due to diffuse bilateral renal involvement patients with
oncocytosis often present with abnormal renal function. Final diagnosis is by biopsy

How well did you know this?
1
Not at all
2
3
4
5
Perfectly
51
Q

A 12-month-old infant with a history of aniridia and nephroblastomatosis undergoes a follow-
up CT of abdomen that demonstrates bilateral enlarged kidneys with a thick rind of
homogenous, non-enhancing, hypodense tissue bilaterally. A focal heterogenous enhancing
mass with cystic change is noted on one side. What is the diagnosis? [B1 Q57]

A. Neuroblastoma.
B. Wilms tumour.
C. Lymphoma.
D. Renal cell carcinoma.
E. Cystic nephroma.

A

Wilms tumour.

Sporadic aniridia is associated with nephron-blastomatosis (multiple nephrogenic rests) and
an increased risk of Wilms tumour. At CT, the nephrogenic rests appear as homogenous,
hypodense nodules/peripheral rind, with little or no enhancement compared to the
compressed normal cortical tissue.

Any new, enlarging, or heterogenous mass in the setting of nephron-blastomatosis indicates
the development of a Wilms tumour.

How well did you know this?
1
Not at all
2
3
4
5
Perfectly
52
Q

A 4-year-old child was referred for a palpable abdominal mass and abdominal pain. Ultrasound
shows a large heterogenous mass in the abdomen. Contrast enhanced CT demonstrates a large
heterogenous and necrotic mass arising from the right kidney, extending across the midline,
and displacing the aorta and inferior vena cava. There are calcifications within the lesion. What
is the most likely diagnosis? [B5 Q31]

(a) Neuroblastoma
(b) Nephroblastoma
(c) Renal cell carcinoma
(d) Oncocytoma
(e) Hepatoblastoma

A

Nephroblastoma

Also called Wilms’ tumour, this lesion typically presents as a large mass, crossing the midline
and commonly displacing the large vessels (in contrast to neuroblastoma, where the mass
encases the vessels). This is the most common neoplasm in children between 1 and 8 years
of age.

How well did you know this?
1
Not at all
2
3
4
5
Perfectly
53
Q

Considering Squamous Cell Carcinoma (SCC) of the kidney: [B3 Q32]

A. Is the second most common tumour subtype affecting the kidney
B. Usually carries a better prognosis than RCC
C. Is usually indolent
D. Acute infection is involved in the aetiology
E. Renal calculi are present in most patients

A

Renal calculi are present in most patients

SCC of the kidney is a relatively rare condition. It carries a poor prognosis due to its aggressive
nature
. Both renal calculi and chronic infection have been implicated in its aetiology. Cross
sectional imaging appearances are identical to those of TCC

How well did you know this?
1
Not at all
2
3
4
5
Perfectly
54
Q

A 3-month-old baby presents with an abdominal lump. Ultrasound shows a large solid mass
arising from the right kidney with focal hypoechoic areas. Contrast enhanced CT shows a solid
right renal mass, involving the renal sinus and multiple small areas of necrosis. There is no
invasion of the renal vein or the collecting system. No metastatic deposits are seen. What is the
most likely diagnosis? [B5 Q44]

(a) Wilms’ tumour
(b) Mesoblastic nephroma
(c) Nephro-blastomatosis
(d) Renal metastases
(e) Lymphoma of kidney

A

Mesoblastic nephroma

This is a hamartoma and is the most common solid neoplasm in neonates. It typically involves
the renal sinus
and there is no invasion of the veins (differentiating it from Wilms’ tumour) or
the collecting system. small areas of necrosis

How well did you know this?
1
Not at all
2
3
4
5
Perfectly
55
Q

A 46-year-old woman has an incidentally discovered large mixed echogenicity mass in the left
flank on an ultrasound examination. A follow-up CT examination is performed, and this shows
a predominantly fatty lesion exophytic to the left kidney, measuring approximately 12 cm in
maximum diameter. Which of the following findings would make the diagnosis of large renal
angiomyolipoma more likely than a perirenal well-differentiated retroperitoneal liposarcoma?
[B1 Q10]

A. The presence of mass effect with displacement of the left kidney.
B. The presence of aneurysmal blood vessels within the lesion.
C. The presence of soft-tissue density areas within the lesion.
D. The presence of ill-defined margins.
E. The presence of fluid density components

A

The presence of aneurysmal blood vessels within the lesion.

Both well-differentiated retroperitoneal liposarcomas and exophytic renal angiomyolipomas
can be very large in size and thus size is not a discriminating factor. Both contain large
amounts of mature lipid, thus significant portions of the lesion will have negative Hounsfield
attenuation values on CT imaging.

Aneurysmal blood vessels are commonly seen in large angiomyolipomas, whereas well
differentiated liposarcomas are generally rather hypo-vascular lesions. Both
angiomyolipomas and retroperitoneal liposarcomas may contain areas of soft tissue density.
Liposarcoma is probably more likely to have an irregular margin and may contain areas of
fluid density

How well did you know this?
1
Not at all
2
3
4
5
Perfectly
56
Q

A 23-year-old female has a renal ultrasound scan for recurrent urinary tract infections. The
only abnormality detected is a 3cm hyperechoic mass in the upper pole of the left kidney. She
subsequently undergoes CT which shows the lesion to have an average HU of–10. Which of
the following is the most likely diagnosis? [B2 Q7]

a. Renal cell carcinoma
b. Transitional cell carcinoma
c. Renal lymphoma
d. Angiomyolipoma
e. Renal abscess

A

Angiomyolipoma

The finding of fat attenuation values within a renal lesion on CT is diagnostic of
angiomyolipoma. This is a benign tumour that is typically hyperechoic on ultrasound and of
high signal on T1-weighted MR due to fat. It does not enhance post-gadolinium, in contrast
to renal cell carcinoma, which usually does enhance

How well did you know this?
1
Not at all
2
3
4
5
Perfectly
57
Q

In a 40-year-old woman with seizures and bilateral renal masses, with the appearance of
multiple angiomyolipomas, which is the single best answer? [B3 Q9]

A. Sporadic solitary Angiomyolipoma (AML) have a female preponderance
B. Account for 50% of all renal AMLs
C. Are associated with tuberous sclerosis
D. Warrant treatment due to risk of bleeding when > 2cm
E. Are typically hypoechoic on US

A

Are associated with tuberous sclerosis

Those AML associated with tuberous sclerosis are multifocal, bilateral, larger and present in
a younger age group. AML appear echogenic on US.

How well did you know this?
1
Not at all
2
3
4
5
Perfectly
58
Q

During MR of the spine a 2 cm renal lesion is identified at the upper pole cortex of the RK.
The lesion is well-defined and is of high SI on T1 and T2 images. Ultrasound shows normal
Doppler flow in the renal vein which is separate from the lesion, which is echogenic on US.
The findings are most likely to represent which of the following? [B3 Q12]

A. AML
B. Atypical cyst
C. Abscess
D. RCC
E. Lymphoma

A

AML

AML appears increased on S1 on T1+T2 and is echogenic on US.

How well did you know this?
1
Not at all
2
3
4
5
Perfectly
59
Q

In a patient who had a right nephrectomy ten years ago for RCC, a 3cm lesion in the cortex of
the lower pole of the left kidney is evaluated with MRI. This appears isointense on the in-phase
sequence and low SI on the opposed phase sequence. No other renal lesions are demonstrated.
The renal veins appear normal. What is the most likely diagnosis? [B3 Q21]

A. AML
B. Simple cyst
C. Oncocytoma
D. RCC
E. Metastatic deposit

A

AML

Demonstration of fat within a renal mass on CT or MRI is diagnostic of AML

How well did you know this?
1
Not at all
2
3
4
5
Perfectly
60
Q

A 60-year-old female who has declined intervention for a renal angiomyolipoma of diameter
6 cm presents with flank pain, hypotension and tachycardia. In this scenario, which of the
following is most likely to account for the presentation? [B4 Q71]

a. torsion of the angiomyolipoma
b. haemorrhage from the angiomyolipoma
c. rupture of the angiomyolipoma
d. leak from the non-aneurysmal abdominal aorta
e. nephroptosis

A

haemorrhage from the angiomyolipoma

Angiomyolipomas are benign hamartomatous tumours that can occur in the kidneys. They
contain fat, smooth muscle and abnormal blood vessels. Eighty per cent are sporadic and
occur most often in females aged 50–80 years. Twenty per cent of patients with
angiomyolipomas have tuberous sclerosis. Retroperitoneal bleed is the most significant
complication and can be catastrophic. The risk increases with size of the lesion due to
increased abnormal vasculature. Haemorrhage occurs because of large tortuous vessels and
aneurysms. Embolization is performed for symptomatic (flank pain) angiomyolipomas, those
that have bled at any size, and prophylactically when over 4cm.

How well did you know this?
1
Not at all
2
3
4
5
Perfectly
61
Q

A 42-year-old man with a history of fits presents with recurrent abdominal pain. Ultrasound
shows a 4 cm heterogenous lesion in the upper pole of the right kidney, with moderate
vascularity. Contrast-enhanced CT shows that the lesion predominantly contains tissue with
Hounsfield units of -65 to -80. Other smaller such lesions were also seen in the left kidney.
What is the most likely diagnosis? [B5 Q25]

(a) Multifocal renal cell carcinoma
(b) Angiomyolipomas
(c) Lymphoma
(d) Metastases
(e) Multiple lipomas

A

Angiomyolipomas

The lesions contain fat (negative Hounsfield units) and show vascularity within. With the
history of fits, tuberous sclerosis should be considered as a diagnosis.

How well did you know this?
1
Not at all
2
3
4
5
Perfectly
62
Q

A 65-year-old male undergoes renal CT following the finding of multiple hypoechoic masses
in both kidneys on ultrasound. Multiple poorly defined masses of decreased attenuation are
demonstrated, which encase the renal vessels. The vessels remain patent, however, and the
renal contour is preserved. Which of the following is most likely to represent the underlying
diagnosis? [B2 Q45]

a. Renal cell carcinoma
b. Transitional cell carcinoma
c. Multiple myeloma
d. Non-Hodgkin’s lymphoma
e. Reninoma

A

Non-Hodgkin’s lymphoma

The most likely diagnosis is non-Hodgkin’s lymphoma. Primary renal involvement is rare but
is often involved either by haematogenous spread or direct invasion. The kidneys represent
one of the most common extra-nodal sites of disease in non-Hodgkin’s lymphoma but are
rarely involved in Hodgkin’s disease. Involvement is usually bilateral, with masses of
decreased attenuation and mild homogenous enhancement with intravenous contrast on CT.
Patency of the renal vessels despite encasement is highly suggestive, as is preservation of the
normal renal contour.

How well did you know this?
1
Not at all
2
3
4
5
Perfectly
63
Q

Regarding lymphoma of the kidneys: [B3 Q29]

A. Multiple focal nodules appear hyperdense on CT
B. Involved kidneys are usually atrophic
C. Focal masses appear high signal on T1
D. Focal masses appear hyperintense on T2
E. CT may demonstrate sheet like diffuse infiltration of perirenal tissues

A

CT may demonstrate sheet like diffuse infiltration of perirenal tissues

Diffuse infiltration leads to renal enlargement. Focal lesions have a characteristic usually low
attenuation post-contrast on CT, low SI on T1 and hypo-iso-intense on T2

How well did you know this?
1
Not at all
2
3
4
5
Perfectly
64
Q

Regarding metastatic sites of disease to the kidney: [B3 Q30]

A. Are frequently symptomatic
B. Usually occurs from direct invasion
C. Haematogenous metastasis are usually > 3 cm
D. Are usually hypo-vascular on CT
E. Commonly calcifies

A

Are usually hypo-vascular on CT

Metastases are usually small (< 3cm), multiple and confined to cortex. The most common
mode of spread is haematogenous. Metastases tend not to invade the renal vein or calcify;
they are more infiltrative, less exophytic compared with renal cell carcinoma

How well did you know this?
1
Not at all
2
3
4
5
Perfectly
65
Q

A 40-year-old man has an ultrasound examination of the abdomen for epigastric pain. The
examination is normal apart for an isoechoic ‘mass’ at the mid-pole of the left kidney. You
suspect this may be a normal variant, such as a prominent column of Bertin. You recommend
a CT scan of kidneys to confirm this. Which phase of a CT renal tract examination would best
show this variant? [B1 Q33]

A. Unenhanced scan.
B. Cortico-medullary phase.
C. Nephrographic phase.
D. Early excretory (3-minute) phase.
E. Late excretory (10-minute) phase

A

Cortico-medullary phase.

Imaging in the cortico-medullary phase (25-40 sec) is helpful for showing the normal cortico-medullary
pattern in pseudotumours such as prominent columns of Bertin or focal renal hypertrophy. It is also useful
for suspected abnormalities such as vascular malformations and pseudoaneurysms of the kidney.

The nephrographic phase (70-180 sec) is best to characterize renal masses. Small parenchymal renal lesions may be hidden in the renal medulla on the cortico-medullary phase and excretory phases.

Excretory phases(>180 sec) are best to assess for collecting system lesions and the unenhanced scan is
best for detection of renal calculi and for using as a baseline in assessing the enhancement
characteristics of parenchymal renal lesions

How well did you know this?
1
Not at all
2
3
4
5
Perfectly
66
Q

A 43-year-old female has a renal ultrasound which shows a left-sided renal ‘mass’. The ‘mass’
is continuous with the renal cortex and has the same echogenicity as the cortex. It is situated at
the border of the upper and mid poles of the left kidney and is seen to extend between the renal
pyramids. Which one of the following are these features most likely to represent? [B2 Q55]

a. Renal scarring
b. Hypertrophied column of Bertin
c. Dromedary hump
d. Persistent fatal lobulation
e. Duplex kidney

A

Hypertrophied column of Bertin

Many lesions may be mistaken for a renal cell carcinoma on imaging, and it is important to be able to differentiate such ‘pseudotumours’ from genuine carcinomas. The features described in the question are consistent with a prominent column of Bertin. This is normal renal tissue located between the pyramids and extending into the renal sinus. The key features include continuity with the cortex, identical echogenicity to normal cortex and the lack of mass effect or renal outline deformity.

  • A dromedary hump is a focal bulge on the lateral border of the left kidney caused by
    its relationship with the adjacent spleen.
  • Persistent fetal lobulation can be identified by indentations of the renal surface that
    overlie the space between the pyramids
  • Renal scarring lies directly over the medullary pyramids
How well did you know this?
1
Not at all
2
3
4
5
Perfectly
67
Q

A 35-year-old man with a facial ‘port-wine stain’ and history of epilepsy presents with
haematuria. Contrast-enhanced CT abdomen shows vascular malformations in the kidney and
spleen. What is the most likely diagnosis? [B5 Q47]

(a) von Hippel–Lindau disease
(b) Sturge–Weber–Dimitri syndrome
(c) Neurofibromatosis type 1
(d) Neurofibromatosis type 2
(e) Tuberous sclerosis

A

Sturge–Weber–Dimitri syndrome

This is characterised by multiple vascular malformations in the face (‘port-wine stain’) and
central nervous system (leptomeningeal venous angiomas), and orbital and visceral
angiomatosis (intestine, kidneys, spleen, thyroid, pancreas, and lungs).

Syndromes with renal mass

  1. Von-Hippel Lindau
  2. Tuberous Sclerosis
  3. Birt Hogg Dube
  4. Sturge-Weber-Dimitri
How well did you know this?
1
Not at all
2
3
4
5
Perfectly
68
Q

A 60-year-old man has an unenhanced CT scan of renal tracts for suspected right renal colic.
The examination is normal apart from an exophytic rounded lesion at the mid-pole of the left
kidney, which is denser than adjacent renal parenchyma. You elect to perform an intravenous
contrast enhanced examination in the nephrographic phase to further evaluate this lesion.
Which of the following Hounsfield attenuation values would be most appropriate if this lesion
is benign? [B1 Q49]

A. 20 pre-contrast attenuation, 30 post-contrast attenuation.
B. 30 pre-contrast attenuation, 60 post-contrast attenuation.
C. 40 pre-contrast attenuation, 50 post-contrast attenuation.
D. 50 pre-contrast attenuation, 80 post-contrast attenuation.
E. 60 pre-contrast attenuation, 70 post-contrast attenuation.
F. 70 pre-contrast attenuation, 100 post-contrast attenuation

A

60 pre-contrast attenuation, 70 post-contrast attenuation.

The attenuation of the normal renal parenchyma typically ranges from 30 to 40 HU. That of
hyperattenuating renal masses usually is at least 40 HU but no higher than 90 HU on CT
without intravenous contrast. Benign cysts are overwhelmingly the most common type of
hyperattenuating renal mass and are also known as hyperdense renal cysts. They are usually
cysts containing haemorrhage or proteinaceous material. Hyperattenuating cysts should not
enhance and therefore cannot be diagnosed with confidence by using unenhanced CT alone.
A proper CT examination includes scanning both before and after the administration of
intravenous contrast material. Masses that increase in attenuation by 10 HU or less are
considered non-enhancing. Masses that increase in attenuation by more than 10 HU are
considered enhancing. However, because the standard deviation of attenuation
measurements may be more than 10 HU, an attenuation difference of 20 HU or more is a
more specific criterion of enhancement

How well did you know this?
1
Not at all
2
3
4
5
Perfectly
69
Q

A 45-year-old man has a complex cyst identified in the right kidney on an ultrasound scan
performed to assess non-specific epigastric pain. He subsequently has a CT scan of kidneys
carried out pre and post administration of intravenous contrast. You classify the complex cyst
as IIF (II requiring follow-up) according to the Bosniak classification. Which of the following
features is most likely seen at CT imaging? [B1 Q59]

A. Presence of calcification.
B. Thickened smooth wall with enhancement.
C. Multiple thin non-enhancing septa.
D. Hyperattenuating cyst <3 cm in size.
E. Thickened irregular wall.

A

Multiple thin non-enhancing septa.

The criteria for a Bosniak IIF cyst are multiple hairline-thin septa with or without perceived (not measurable) enhancement, minimal smooth thickening of wall or septa that may show perceived (not measurable) enhancement, calcification may be thick and nodular but no measurable enhancement present, no enhancing soft-tissue components, and intrarenal non-enhancing high attenuation renal mass (> 3cm). The follow-up is CT or MRI at 6 months, 1 year, and then yearly up to 5 years.

The criteria for a Bosniak II cyst that does not require follow-up include few hairline-thin septa
with or without perceived (not measurable) enhancement, fine calcification or a short
segment of slightly thickened calcification in the wall or septa, homogeneously high-
attenuating masses (<3 cm) that are sharply marginated and do not enhance.

Thickened irregular or smooth walls or septa, with measurable enhancement, are features of
a Bosniak III cyst and these are surgical lesions unless there are co-morbidities or limited life
expectancy, when they may be observed

How well did you know this?
1
Not at all
2
3
4
5
Perfectly
70
Q

A 71-year-old male undergoes renal CT for characterisation of a cystic renal mass. Which one
of the following five features would classify the lesion as a Bosniak III lesion? [B2 Q21]

a. Lack of enhancement
b. Septation
c. Minimally irregular wall
d. Curvilinear calcification
e. Uniform wall thickening

A

Uniform wall thickening

The Bosniak classification groups cystic renal lesions into one of four categories based on
CT/MR appearances. The differentiation between groups II and III is important as group II are
typically ‘follow-up lesions and group III are ‘surgical lesions. Features of a Bosniak III lesion
include irregular thickened septa, measurable enhancement, coarse irregular calcification,
multi-loculation, nodularity, uniform wall thickening and margin irregularity

How well did you know this?
1
Not at all
2
3
4
5
Perfectly
71
Q

A 3 cm cystic lesion is seen on CT. Thickened septae are noted, with nodular areas of
calcification, with solid non-enhancing areas. Which of the following Bosniak classification
best describes the lesion? [B3 Q27]

A. I
B. II
C. IIF
D. III
E. IV

A

III
Class II have at least one thin septa traversing them (< 1mm) and they have an appearance of
thin areas of mural calcification or fluid content with greater attenuation. These lesions are
benign however IIF with numerous class II features should be followed up. Class III features
as above are indicative of malignancy and biopsy, or surgical exploration is necessary. Type IV
cystic lesions are clearly malignant.

How well did you know this?
1
Not at all
2
3
4
5
Perfectly
72
Q

A 55-year-old male has an ultrasound scan of the renal tract prompted by a single urinary tract
infection. A kidney cyst of diameter 2 cm with a thin septum is seen. The septum has
perceptible enhancement on CT. What is the most appropriate management from the choices
below? [B4 Q4]

a. discharge with no follow-up
b. imaging follow-up
c. partial nephrectomy
d. nephrectomy
e. nephroureterectomy

A

Imaging follow-up

An incidental, mildly complicated renal cyst has been uncovered. The Bosniak classification is
a useful tool for evaluating cystic renal lesions, and guiding management. Simple cysts
(Bosniak grade I) are thin walled, are of water density and have no enhancement. Minimally
complicated cysts (grade II) may be clustered or septated, and have small curvilinear
calcifications, a minimally irregular wall or high-density contents. Follow-up lesions (grade IIF)
have perceptible enhancement of otherwise thin septations or are above 3cm in diameter
with high-density contents. Surgical lesions (grade III) have thicker septa or walls, measurable
enhancement, coarse irregular calcification and irregular margin, are multiloculated or can be
a non-enhancing nodular mass. Clearly malignant lesions (grade IV) can have necrotic
components, irregular wall thickening and enhancing solid elements

How well did you know this?
1
Not at all
2
3
4
5
Perfectly
73
Q

A 50-year-old man is discovered to have a cystic abnormality at the lower pole of the left
kidney during an ultrasound scan performed for right upper quadrant pain. You think this might
be a complex cystic lesion and therefore recommend a CT scan of renal tracts. This shows a
conglomeration of variable sized cysts at the lower pole of the left kidney, but no capsule or
mural irregularities of the cysts. What is the most likely diagnosis? [B1 Q72]

A. Localized cystic renal disease.
B. Multilocular cystic nephroma.
C. Renal lymphangiomatosis.
D. Cystic clear cell carcinoma.
E. Multicystic dysplastic kidney

A

Localized cystic renal disease.

This is an uncommon, non-familial, and non-progressive disorder of the kidney characterized
by the replacement of all or localized areas of a kidney by multiple variably sized cysts. These
cysts form clusters that are separated by thin areas of normal renal parenchyma. The
aggregated cysts in localized cystic disease can frequently appear like a multi-septate mass,
but they do not form a distinct encapsulated mass and do not show mural irregularities, which
are characteristics of cystic neoplasms.

Multilocular cystic nephroma in adults is much more common in females (female:male 9:1)
and manifests as a multi-loculated cystic lesion with hair-like septa and minimal mural
enhancement. It is distinguished from localized cystic disease by the presence of a capsule.
Typically, extension into the central renal sinus is found and multilocular cystic nephroma is
often classified on imaging as Bosniak III lesions.

Lymphangioma of the kidney is a rare benign cystic tumour that most often arises from the
peri-pelvic region or renal sinus. It may more rarely arise from the lymphatics of the capsule
or cortex. In the diffuse form of lymphangiomatosis, the cystic changes occur diffusely in the
renal sinus or perinephric region, with a relatively normal appearing renal parenchyma.

Clear cell RCC presents as a solid or a cystic lesion, the cystic variant accounting for 4–15% of
all RCCs. These lesions typically have nodular or septal enhancement, distinguishing them
from benign cystic renal lesions.

Multi-cystic dysplastic kidney is a congenital maldevelopment in which the kidney is
completely replaced by cysts with no normal renal parenchyma remaining.

How well did you know this?
1
Not at all
2
3
4
5
Perfectly
74
Q

A 50-year-old female undergoes CT for an echogenic renal lesion on ultrasound. Pre-contrast
CT shows a lesion, which is well-defined and has increased attenuation (60 HU). Postcontrast
the lesion appears low in attenuation related to the surrounding parenchyma (61 HU). The
diagnosis is: [B3 Q47]

A. Haemorrhagic renal cell carcinoma
B. Angiomyolipoma (AML) that has bled
C. Lymphoma
D. Haemorrhagic renal cyst
E. AML

A

Haemorrhagic renal cyst

A less than 10 HU increase post-contrast indicates benign hyperdense cyst. Other benign
features include being sharply marginated and homogenous

How well did you know this?
1
Not at all
2
3
4
5
Perfectly
75
Q

A 60-year-old male has an ultrasound scan of the renal tract for renal colic. There is an echo-
free, thin-walled structure in the renal sinus with posterior acoustic enhancement and dilatation
of the major calyces. A CT KUB (unenhanced) does not add to this appearance, but a 10-minute
delayed, contrast-enhanced CT shows that the calyces are obstructed while the renal pelvis is
not dilated but stretched over the non-enhancing sinus abnormality. What is the most likely
diagnosis? [B4 Q50]

a. sinus lipomatosis
b. parapelvic cyst
c. pelvi-ureteric junction obstruction
d. transitional call carcinoma
e. renal cell carcinoma

A

Parapelvic cysts

The main differential diagnosis for a parapelvic or renal sinus cyst is hydronephrosis. Such
cysts may present with pain due to obstructive caliectasis, but rarely cause hydronephrosis.
They are found in 1.5% of autopsies and represent 4–6% of all renal cysts. A distinction is
made in this question between renal and ureteric colic, the former symptom located to the
loin and the latter more typically being loin to groin

76
Q

A 40-year-old male with a history of haematuria undergoes CT urography. Initial non-contrast
scan demonstrates right-sided medullary nephrocalcinosis. Following intravenous contrast
administration, a striated ‘paintbrush’ appearance of the renal medulla is noted. The left kidney
is unremarkable. What is the diagnosis? [B1 Q36]

A. Hyperparathyroidism.
B. Renal tubular acidosis.
C. Medullary sponge kidney.
D. Sarcoidosis.
E. Multiple myeloma

A

Medullary sponge kidney.

Hyperparathyroidism, renal tubular acidosis, and medullary sponge kidney are the three most
common causes of medullary nephrocalcinosis. The former two conditions are associated
with hypercalciuria that results in uniform medullary nephrocalcinosis. Sarcoidosis and
multiple myeloma are associated with hypercalcemia resulting in bilateral nephrocalcinosis.
Medullary sponge kidney can affect the kidney segmentally, unilaterally, or bilaterally,
therefore unilateral nephocalcinosisis is suggestive of medullary sponge kidney.

Medullary sponge kidney is characterized by cystic dilatation of collecting tubules. Urine stasis
within the dilated tubules predisposes to infection and calculus formation within the dilated
tubules or urinary tracts. On excretory urogram, contrast within the dilated tubules produces
a striated ‘paintbrush’ appearance of the renal pyramids
.

77
Q

A 21-year-old female undergoes a renal ultrasound scan following an abdominal X-ray that
demonstrated multiple foci of calcification in both renal areas. The ultrasound reveals multiple
medullary cysts bilaterally which are seen to communicate with the collecting system. The
medullae of both kidneys are of increased echogenicity. Which of the following diagnoses is
most likely? [B2 Q44]

a. Mega-calicosis
b. multi-cystic dysplastic kidney
c. Autosomal dominant polycystic kidney disease
d. Autosomal recessive polycystic kidney disease
e. Medullary sponge kidney

A

Medullary sponge kidney

The features are those of medullary sponge kidney. This is a non-inheritable condition that
produces cystic dilatation of the collecting ducts and nephrocalcinosis; 75% of cases are
bilateral and it is usually asymptomatic. It is associated with an increased incidence of
infection and urolithiasis but is not thought to predispose to malignancy

78
Q

Which is the most likely to cause bilateral small rather than large kidneys? [B3 Q43]

A. Medullary cystic disease
B. Multiple myeloma
C. PAN
D. Glycogen storage disease
E. Acute glomerulonephritis

A

Medullary cystic disease

Acute arterial hypotension, arteriosclerosis, nephrosclerosis and hereditary nephropathies
including medullary cystic disease and Alport syndrome are all causes of bilateral small
kidneys.

79
Q

An 18-year-old male presented with left ureteric colic. Intravenous urogram shows that both
the kidneys are enlarged and there is elongation, displacement, and deformity of the calices
(spider leg appearance). What is the most likely diagnosis? [B5 Q18]

(a) Lymphoma
(b) Autosomal dominant polycystic kidney disease
(c) End-stage renal failure
(d) Renal vein thrombosis
(e) Autosomal recessive polycystic kidney disease

A

Autosomal dominant polycystic kidney disease

Intravenous urogram appearances are typical due to large cysts distorting the collecting
system. Spider leg pyelogram - Stretched out collecting system due to mass effect from renal cysts

Autosomal recessive polycystic kidney disease (infantile type) produces microscopic cysts and
on intravenous urography and shows an initial faint nephrogram with increasingly dense
nephrogram.

80
Q

A 20-week routine anatomy scan of a pregnant woman shows oligohydramnios. The kidneys
were hyperechoic and enlarged with a renal: abdominal circumference ratio of 0.35. The
urinary bladder was empty. What is the most likely diagnosis? [B5 Q43]

(a) Autosomal recessive polycystic kidney disease
(b) Autosomal dominant polycystic kidney disease
(c) Posterior urethral valves
(d) von Hippel–Lindau disease
(e) Nephroblastomatosis

A

Autosomal recessive polycystic kidney disease

This condition can be diagnosed as early as 17–18 weeks on obstetric ultrasound. The kidneys
may be massively enlarged measuring up to 10–20 times normal size with an enlarged renal:
abdominal circumference ratio of > 0.30. The renal parenchyma is hyperechoic and there may
be oligohydramnios. There is an association with congenital hepatic fibrosis.

81
Q

A 50-year-old man of Indian origin was involved in a road traffic accident. CT scan of the
abdomen shows a small non-functional shrunken kidney with extensive dystrophic
calcifications. What is the most likely diagnosis? [B5 Q4]

(a) Renal tuberculosis
(b) Chronic renal failure
(c) Lymphoma
(d) Renal cell carcinoma
(e) Multi-cystic dysplastic kidney

A

Renal tuberculosis

This appearance is typical of a ‘putty kidney’. This results in a non-functional, shrunken kidney
and auto-nephrectomy. Tuberculosis can involve any part of the genitourinary tract. Renal
tuberculosis is usually seen secondary to haematogenous spread from lungs. Renal
tuberculosis can manifest in several ways including as a renal mass

82
Q

A 54-year-old female patient with diabetes presents with a history of left loin pain, pyrexia,
and pyuria over the past 24 hours. The patient develops sepsis soon after admission and is
brought for a CT scan following appropriate resuscitation. The CT shows a mildly dilated left
renal pelvis, which contains small pockets of air extending into the calyces. A small wedge-
shaped area of poor enhancement in the lower pole is the only parenchymal abnormality. The
clinical diagnosis is of pyelonephritis, but what subtype of pyelonephritis is best represented
by these imaging features? [B1 Q15]

A. Acute pyelonephritis.
B. Emphysematous pyelitis.
C. Emphysematous pyelonephritis
D. Acute pyonephrosis.
E. XGP.

A

Emphysematous pyelitis.

The combination of these radiological findings in a septic diabetic patient indicates an
infection with a gas-forming organism, usually E. coli. The gas is located entirely within the
pelvicalyceal system, as opposed to the renal parenchyma, which would indicate a diagnosis
of emphysematous pyelonephritis. The importance is in the urgency of treatment and
prognosis. Emphysematous pyelonephritis carries a grave prognosis and often requires
nephrectomy for life-saving treatment, whereas the prognosis with emphysematous pyelitis
is not as grave and it often responds to antibiotics with possible urinary drainage procedures
if necessary.

83
Q

E. coli, with or without an underlying diagnosis of diabetes mellitus, is the most common
pathogen behind many urological conditions. All of the following conditions are most
commonly due to E. coli infection except one, which one? [B1 Q1]

A. Xanthogranulomatous pyelonephritis (XGP).
B. Emphysematous pyelonephritis.
C. Pyonephrosis.
D. Pyeloureteritis cystica.
E. Malakoplakia

A

Xanthogranulomatous pyelonephritis (XGP).

This is most commonly secondary to Proteus infection. XGP is discussed elsewhere in this
chapter.

84
Q

A 64-year-old female patient presents with a 3-month history of right-sided loin pain,
microscopic haematuria, and low-grade pyrexia. Her GP had been treating the patient as a
presumptive UTI, given the background history of diabetes, but is concerned that the symptoms
are not resolving. On clinical examination the urologists have felt a right-sided ballotable loin
mass. They have requested a CT scan of abdomen. This reveals a large irregular mass replacing
most of the parenchyma of the right kidney, which measures 10 × 11 cm in axial diameter and
does not demonstrate significant enhancement. There are several focal masses noted within the
dominant mass that have attenuation values of between –15 and –5 HU. The renal outline is
poorly defined, with involvement of the perirenal fascia. A large staghorn calculus is also
present. There is no evidence of invasion of the renal vein. What is the primary diagnosis?

A. Renal cell carcinoma.
B. Renal oncocytoma.
C. Renal lymphoma.
D. XGP.
E. Renal angiomyolipoma

A

XGP.

Whilst the patient presents with the classical renal cell carcinoma (RCC) triad of loin pain,
haematuria, and a mass, this triad is actually only present in 30% of patients with RCC. These
features are also typically found in XGP. The key finding in this case is of areas of fatty
attenuation within the mass, in association with a staghorn calculus
. Calculi are found in 80–
90% of cases of XGP. The long history of symptoms is also typical of XGP, which can be present
for up to 6 months prior to diagnosis. Lymphoma, RCC, and renal oncocytoma would not
typically have areas of fatty tissue within them. Angiomyolipomas, in the absence of tuberous
sclerosis, are typically solitary. They are usually well defined, unlike in this case, and would
rarely reach this size without having previously caused symptoms. XGP is typically found in
diabetic patients and is due to proteus infection. Treatment is with nephrectomy

85
Q

A 56-year-old female presents with a three-month history of pyrexia, loin pain and weight loss. Urinalysis reveals pyuria and haematuria. Urinary culture reveals Proteus mirabilis. A renal CT demonstrates a globally enlarged kidney with extensive perirenal inflammation, an absent nephrogram and a staghorn calculus. Which one of the following diagnoses is most likely? [B2 Q31]

a. Leukoplakia
b. Emphysematous pyelonephritis
c. Pyeloureteritis cystica
d. Xanthogranulomatous pyelonephritis
e. Page kidney

A

Xanthogranulomatous pyelonephritis

The most likely diagnosis is xanthogranulomatous pyelonephritis. This is a chronic
granulomatous infection in a chronically obstructed kidney, often secondary to a staghorn
calculus. It often presents insidiously in middle-aged to elderly females. It is most commonly
diffuse, but the focal form seen in 15% of cases may provide a diagnostic dilemma as it can
be difficult to confidently distinguish from renal cell carcinoma.

86
Q

A 45-year-old female with a six-month history of urinary tract symptoms undergoes CT
showing an enlarged right kidney with a large staghorn calculus and hydronephrosis. The renal
parenchyma is replaced by multiple confluent fluid-filled masses. Open nephrectomy histology
from the right kidney reveals lipid laden foamy macrophages in combination with an
inflammatory granuloma and a lymphoplasmocytic infiltrate. What is the most likely diagnosis?
[B3 Q16]

A. Xanthogranulomatous pyelonephritis
B. Renal lymphoma
C. Renal TB
D. Malakoplakia
E. Nephrocalcinosis

A

Xanthogranulomatous pyelonephritis

A rare form of low-grade chronic renal infection with progressive destruction of renal
parenchyma, XGP is more common in women and in 50-60 age group. Most cases are diffuse.
E.coli and P.mirabils are the most common organisms in these patients with UTI

87
Q

An 80-year-old male has an IVU for unilateral loin pain. On the control film, the renal outline
on the side of the pain is indistinct and enlarged. The same kidney has a staghorn calculus and
shows no evidence of function following contrast injection. Which of the following is the most
likely diagnosis? [B4 Q15]

a. xanthogranulomatous pyelonephritis
b. renal tuberculosis
c. hyperparathyroidism
d. hydatid cyst
e. carcinoma

A

Xanthogranulomatous pyelonephritis

On the IVU, the features of xanthogranulomatous pyelonephritis are unilateral reniform
enlargement, ipsilateral renal hypofunction and nephrolithiasis. The condition is probably
produced by chronic low-grade obstruction and chronic bacteriuria. Renal parenchyma is
replaced by lipid-laden histiocytes causing renal expansion. This expansion can cause ‘stone
fracture’ with obvious separation of the fracture fragments on plain film. Renal tuberculosis
causes dystrophic calcification that can be nodular, curvilinear, or amorphous. The
calcification is typically multifocal, involving other sites of the urinary tract, and there may be
kidney scarring. Hyperparathyroidism causes medullary calcification. Six per cent of renal
carcinomas have amorphous, irregular, or occasionally curvilinear calcification, while hydatid
cysts exhibit curvilinear or heterogeneous calcification.

88
Q

A 55-year-old woman presents to her general physician with left renal colic and chronic
recurrent urinary tract infections. CT of the urinary tract shows a staghorn calculus in the left
kidney. Post-contrast images demonstrate multiple low attenuation masses, almost replacing
the renal parenchyma of left kidney with peripheral rim enhancement, and minimal contrast
excretion from the left kidney on delayed phase images. What is the most likely diagnosis? [B5
Q28]

(a) Hydronephrotic kidney
(b) Renal cell carcinoma
(c) Xanthogranulomatous pyelonephritis
(d) Pyonephrosis
(e) Lobar nephronia

A

Xanthogranulomatous pyelonephritis

This is a chronic suppurative infection characterised by replacement of renal parenchyma by
lipid-laden macrophages. In most cases there is also a staghorn calculus with renal
enlargement and a rim of cortical enhancement (‘bear paw’ sign)

89
Q

A 50-year-old man with an underlying condition presents with acute abdominal pain. CT shows
haemorrhage involving the left kidney. What is the most unlikely diagnosis? [B5 Q30]

(a) Xantho-granulomatous pyelonephritis
(b) Renal cell carcinoma
(c) Multiple angiomyolipoma
(d) Polyarteritis nodosa
(e) Cortical cysts

A

Xanthogranulomatous pyelonephritis

Other options are all associated with bleeding or spontaneous haemorrhage.

90
Q

A 24-year-old male patient presents to the A&E department with a history of severe episodic
right-sided loin pain, radiating to the groin. He has no history of previous renal calculi. A low-
dose non-contrast CT of the renal tracts shows a calculus in the distal right ureter, adjacent to
the vesico-ureteric junction. The calculus measures 7 mm in diameter. The patient has a
horseshoe kidney. There is stranding in the peri-nephric fat and around the right ureter. The
density of the calculus is measured to be 1500 HU. Which of these observations is least likely
to have relevance to this patient’s treatment? [B1 Q50]

A. Site of the calculus.
B. Size of the calculus.
C. Perinephric and periureteric stranding.
D. Density of the calculus.
E. Horseshoe kidney

A

Perinephric and periureteric stranding.

Whilst this observation is sensitive and specific for detecting calculi, it does not have a bearing
on stone outcome or urgency of treatment. The site is of value because if the stone impacts,
it dictates the therapeutic approach. The size of the calculus can be used to assess the
likelihood of spontaneous passage, with calculi over 5 mm more likely to cause obstruction.
The density of the calculus has been found to predict the success of extracorporeal shock
wave lithotripsy (ESWL). The presence of anatomical variations is also of interest to the
urologists in treatment planning, if this becomes necessary.

91
Q

A 25-year-old male patient presents to A&E with left renal angle pain, radiating to the groin
and significant haematuria on dipstick urinalysis. His kidneys, ureters, and bladder (KUB) x-
ray is normal. He has a CT KUB, which shows no evidence of a calculus, but a mildly dilated
left renal system as far as the mid ureter with peri-ureteric stranding to this point. Given the
classical history, the patient undergoes an MR urogram, which shows a filling defect in the mid
left ureter and a diagnosis of an obstructing ureteric calculus is made. The patient is observed,
and this diagnosis is confirmed when the patient passes a calculus the following day. The
patient has a relevant past medical history, which led you to suspect a calculus despite the CT
findings. Which of these clinical histories would lead you to suspect this? [B1 Q60]

A. Recurrent proteus infection.
B. HIV-positive patient on antiretroviral therapy.
C. Dehydrated patient.
D. Familial history of xanthine calculi.
E. Gout.

A

HIV-positive patient on antiretroviral therapy.

Whilst CT KUB has 95–99% sensitivity for renal calculi, occasionally calculi due to antiretroviral
therapy can be undetectable even to CT. Proteus is associated with formation of staghorn calculi and chronic dehydration with calcium
oxalate calculi, both of which are radio opaque. Xanthine calculi and urate calculi are
classically not detectable on plain film KUB but are usually visible on CT KUB.

92
Q

A 29-year-old male has microscopic haematuria and symptoms suggesting left ureteric colic.
An unenhanced CT abdomen and pelvis is requested. A 4 mm calcific density is seen near the
bladder in the left hemi-pelvis. Which one of the following signs may be useful to help
differentiate between a phlebolith and ureteric calculi? [B2 Q33]

a. Lobster claw sign
b. Soft-tissue rim sign
c. Signet ring sign
d. Nubbin sign
e. Drooping lily sign

A

Soft-tissue rim sign

The soft-tissue rim sign is thickening of the ureteric wall around the calculus due to oedema.
It has a reported specificity of up to 92% for renal calculi. Other signs that may favour a
diagnosis of ureteric calculi include asymmetrical perinephric fat stranding, periureteral
oedema, hydronephrosis and unilateral renal enlargement.

The lobster claw and signet ring signs concern papillary necrosis on intravenous urogram.

The nubbin and drooping lily signs both refer to ureteral duplication.

93
Q

A 60-year-old patient with a history of previous urinary tract interventions presents with right
hydronephrosis and deranged renal function. Imaging suggests a mid-ureteric stenosis. An MR
urogram is planned. Which of the following is true? [B2 Q53]

a. A static-fluid MR urography is performed using gadolinium-enhanced T1-weighted imaging
b. Excretory MR urogram is preferred in patients with severe renal impairment
c. T1-weighted imaging is useful in differentiating between clot and calculi
d. Renal sinus cyst can be differentiated from dilated intrarenal collecting system on T1-
weighted imaging
e. smaller filling defects are better seen on the MIP images rather than the source data

A

T1-weighted imaging is useful in differentiating between clot and calculi

  • A static-fluid MR urogram is a heavily T2-weighted technique like MRCP.
  • Excretory urogram is a post-gadolinium injection T1-weighted technique.
  • A static-fluid MR urogram is preferred in patients with renal failure.
  • Renal sinus cysts cannot often be differentiated from dilated intrarenal collecting systems
    on T1-weighted and T2-weighted images. They are better appreciated on the excretory
    urogram. Source data should be reviewed to ensure that small filling defects are not
    missed.
94
Q

A 47-year-old with an obstructed urinary system is advised to have a percutaneous
nephrostomy. Which of the following is appropriate? [B2 Q54]

a. Persistent post-procedural haematuria usually needs a nephrectomy
b. If appropriate, the preferred site of puncture on the renal surface is just anterior to the convex
lateral margin
c. A lower pole calyx is preferred when ureteral intervention is planned
d. There is a 10% chance of developing haematuria post-procedure
e. In an obstructed infected system, further imaging and manipulation are usually delayed after
establishing drainage

A

In an obstructed infected system, further imaging and manipulation are usually delayed after
establishing drainage

  • The most common reasons for persistent haematuria are traumatic arteriovenous fistula,
    pseudoaneurysm or vascular injury, all of which are usually managed endovascularly.
  • Brodel’s avascular plane is just posterior to the convex lateral margin.
  • Whilst an easily accessible lower pole calyx is usually the target for a simple nephrostomy
    drainage, for ureteral interventions, a posterior calyx in the mid or upper polar region may
    be better.
  • Almost all patients develop haematuria, but 1–3% may need transfusion or further
    intervention.
95
Q

Which is a cause of dilated calyx with wide infundibula? [B3 Q11]

A. Post-obstructive uropathy
B. Structure secondary to calculus
C. Structure secondary to TB
D. Extrinsic compression by an artery
E. Hydrocalycosis

A

Post obstructive uropathy

Answers B-E are causes with narrow infundibula.

96
Q

A 38-year-old man presents with a classic history of ureteric colic. Plain abdominal film is
unremarkable, and CT KUB shows ureteric dilatation and periureteric stranding down to the
vesico-ureteric junction on the side of the pain. No radio-opaque calculi are seen on the CT
scan. Ultrasound examination shows a tiny, densely echogenic focus within the bladder wall,
at the same vesico-ureteric junction. For which of the following conditions is the patient most
likely to be receiving treatment? [B4 Q40]

a. diabetes mellitus
b. asthma
c. HIV
d. gastro-oesophageal reflux
e. headaches

A

HIV

While around 10% of renal and ureteric stones are radiolucent on plain film, almost all are
opaque on CT. One exception is the tiny radiolucent calculi formed in patients on protease
inhibitors such as indinavir used in the treatment of HIV/AIDS.

97
Q

A 70-year-old man with previously diagnosed bilateral renal calculi is seen with fever, rigors,
and left loin pain. Ultrasound scan shows a dilated left renal collecting system. Which of the
following is the most appropriate management? [B4 Q81]

a. oral antibiotics with outpatient follow-up
b. urinary catheter, intravenous antibiotics, and admission to hospital
c. left nephrostomy and antibiotics
d. bilateral nephrostomy
e. intramuscular anti-spasmodic

A

Left nephrostomy and antibiotics

Pyonephrosis secondary to an obstructing calculus is the likely diagnosis. Percutaneous
nephrostomy is indicated for temporary or permanent relief of an obstructed urinary system
(malignant or benign obstructive uropathy), pyonephrosis, renal stones, iatrogenic ureteric
injury, transplant kidney ureteric obstruction, and vesicovaginal fistula. A ‘onestab’ puncture
technique or Seldinger technique can be used. For any procedures where urinary infection is
suspected or in stone disease, prophylactic antibiotics are mandatory – for example, 80mg
gentamicin or 750mg cefuroxime. Clotting must also be checked and if necessary corrected.
The major complication rate is around 3% while minor complications occur in around 15%.
Major complications are septicaemia, blood loss requiring transfusion, pleural or abdominal
viscera puncture, or transcolonic approach. Minor complications include retroperitoneal
urine extravasation, clot colic from macroscopic haematuria and tube complications. Tube
complications include catheter dislodgement, blockage, leaking, kinking and fracture

98
Q

A patient who has no function in their native kidneys is found to have declining renal function
1 day after transplantation. A MAG3 renogram shows normal perfusion but diminished
excretion. Which of the following processes is affecting the transplanted kidney? [B5 Q1]

a. acute rejection
b. chronic rejection
c. acute tubular necrosis
d. renal vein thrombosis
e. ciclosporin toxicity

A

Acute tubular necrosis

Acute tubular necrosis is the commonest acute reversible cause of renal failure in the
transplanted kidney and usually occurs within 24 hours. Of the complications of a
transplanted kidney causing renal impairment, normal perfusion is seen in acute tubular
necrosis
, whereas renal vein thrombosis and transplant rejection have reduced perfusion
accompanying the diminished excretion
. Ciclosporin can cause a similar pattern of renal
impairment but would be expected to occur 1 month after transplantation.

Functional assessment of a transplanted kidney involves perfusion and excretion assessment
with a MAG3 or DTPA renogram, MAG3 being the better test in transplant recipients with
renal impairment. Doppler ultrasound resistive index measurement is also used, with a value
of ,0.7 regarded as normal.

99
Q

A 38-year-old patient is referred for an urgent IVU and ultrasound by their GP, who has picked
up mild renal impairment on recent blood tests. The ultrasound shows two normal-sized
kidneys with no evidence of cortical loss. The IVU shows a normally enhancing renal outline,
but the pelvicalyceal system is abnormal. Some calyces show a non-specific clubbed or blunted
appearance. Thin tracks are noted extending from other calyces. In a number of calyces there
is a filling defect noted within a rounded, blunted calyx. From these appearances you can make
a diagnosis. The patient has a classical history for this condition. From the given options, what
is the most typical history? [B1 Q65]

A. Recent history of acute hypotension on a background of dehydration.
B. Pyrexia, renal angle pain, and urine culture positive for E. coli.
C. Chronic non-steroidal anti-inflammatory drug (NSAID) analgesic overuse.
D. Previously diagnosed reflux nephropathy.
E. History of TCC within bladder.

A

Chronic non-steroidal anti-inflammatory drug (NSAID) analgesic overuse.

The appearances described are those of renal papillary necrosis. This can be due to several causes, most commonly analgesic abuse, but also severe renal infection in diabetics, sickle cell disease, haemophilia, and renal vein thrombosis.

The causes of chronic renal disease can be subdivided by the effects noted on the renal parenchyma and the papillary/pelvicalyceal system, and whether these are uni- or bilateral. In bilateral cases such as this, no radiological abnormality suggests possible glomerulonephritis, acute tubular necrosis (ATN), acute cortical necrosis, or pyelonephritis.

Generalized infiltration of the renal parenchyma suggests amyloid or malignant infiltration (e.g. lymphoma or leukaemia). Papillary/pelvicalyceal abnormalities, in the presence of normal parenchyma, indicate papillary necrosis, medullary sponge kidney, or renal TB. Papillary calyceal abnormality with focal parenchymal loss indicates reflux nephropathy, TB, or calculus. Both parenchymal loss and papillary calyceal abnormalities indicate obstructive nephropathy or severe reflux nephropathy.

100
Q

A 32-year-old male presents with right flank pain and an intravenous urogram is requested with
the provisional diagnosis of ureteric calculi. The renal outline is smooth and wavy, with a
decreased overall size. The fornixes are widened with club-shaped calyces. After further
questioning he reveals a recent over-use of analgesia. Which one of the following diagnoses is
most likely? [B2 Q32]

a. Acute cortical necrosis
b. Acute tubular necrosis
c. Papillary necrosis
d. Acute interstitial nephritis
e. Haemorrhagic cystitis

A

Papillary necrosis

Papillary necrosis occurs due to ischaemic damage to the medulla of the kidney and does not
primarily involve the cortex. There are many causes including diabetes, analgesic
nephropathy, pyelonephritis, renal vein thrombosis and sickle cell disease. It may be localised
or diffuse, bilateral or unilateral depending on the cause. Intravenous urogram appearances
are varied and include clubbed calyces, calcification and sloughing of necrotic papilla and
alteration in the renal contour.

101
Q

A 30-year-old male is investigated by renal tract ultrasound scan for renal impairment. Both
kidneys are smooth in outline but enlarged. Which of the following diagnoses typically
produces this pattern of renal enlargement? [B4 Q17]

a. autosomal dominant polycystic kidney disease
b. von Hippel–Lindau disease
c. sickle cell disease
d. metastases
e. nephroblastomatosis

A

Sickle Cell Disease

Causes of bilateral smooth renal enlargement include diabetic nephropathy, acute
glomerulonephritis, collagen vascular disease, vasculitis, AIDS nephropathy, leukaemia,
lymphoma, autosomal recessive polycystic renal disease, acute interstitial nephritis, sickle cell
disease, thalassaemia, acromegaly, amyloidosis, myeloma, and acute urate nephropathy.
When the bilateral renal enlargement is caused by masses, the differential diagnosis includes autosomal dominant polycystic disease, acquired renal cystic disease, lymphoma, metastases,
Wilms’ tumours, tuberous sclerosis, von Hippel–Lindau syndrome, multiple oncocytomas and
nephroblastomatosis.

102
Q

An adult male patient who has been taking over-the-counter analgesics regularly for years has
an IVU for ureteric colic. No radiopaque calculi are seen on the control film. With contrast in
the renal calyces, they are noted to be club shaped on the side of the pain. On the same image,
there is a triangular filling defect in the renal pelvis. The colic is most likely to be caused by
which of the following? [B4 Q38]

a. radio-opaque stone
b. radiolucent stone
c. sloughed papilla
d. blood clot
e. transitional cell carcinoma

A

Sloughed papilla

Predisposing factors to papillary necrosis include diabetes mellitus, analgesic
nephropathy/abuse, sickle cell disease, pyelonephritis, obstructive uropathy, tuberculosis,
trauma, cirrhosis, coagulopathy and renal vein thrombosis. None of the other options easily
explains the club-shaped calyx. Blood clots from renal or tumour haemorrhage do cause
ureteric colic but tend to elongate along the pelvis and ureter

103
Q

A 60-year-old diabetic had a contrast–enhanced CT of the chest. He suffered mild skin
eruptions and itching after the scan. The next day, he presented to the Accident & Emergency
Department with abdominal pain and anuria. Plain abdominal radiograph shows smooth large
kidneys with dense bilateral nephrogram and absence of contrast in the collecting system. What
is the most likely diagnosis? [B5 Q20]

(a) Acute tubular necrosis
(b) Acute glomerulonephritis
(c) Acute cortical necrosis
(d) Papillary necrosis
(e) Pyelonephritis

A

Acute tubular necrosis

This is secondary to temporary marked reduction in tubular blood flow. Contrast injection
especially in diabetics with glomerulosclerosis is one of the known causes. Typically, the
kidneys are enlarged due to interstitial oedema with an immediate dense and persistent
nephrogram with absence of contrast from the collecting system.

104
Q

A 31-year-old female is admitted to hospital with placental abruption. Her renal function
deteriorates significantly and therefore a renal ultrasound is requested. Kidneys with bilateral
increased echogenicity and thin tramline calcification of the cortices are seen. Which of the
following underlying conditions is most likely? [B2 Q56]

a. Acute cortical necrosis
b. Papillary necrosis
c. Barter syndrome
d. Drug-related nephrotoxicity
e. Renal infarction

A

Acute cortical necrosis

The features described are typical of cortical nephrocalcinosis. Causes of cortical
nephrocalcinosis include acute cortical necrosis, chronic glomerulonephritis, sickle cell
disease, Alport syndrome and congenital oxalosis. Acute cortical necrosis is a rare cause of
acute renal failure and is most commonly due to complications of pregnancy such as placental
abruption, infected abortion, and severe eclampsia.

105
Q

```

~~~

Which is a cause of cortical rather than medullary nephrocalcinosis? [B3 Q33]

A. Acute cortical necrosis
B. Hyperoxaluria
C. Hypervitamosis D
D. Sarcoidosis
E. Renal tubular acidosis

A

Acute Cortical Necrosis

Acute cortical necrosis causes cortical nephrocalcinosis, whereas B-E are causes of medullary
nephrocalcinosis.

106
Q

A 40-year-old woman presents with nausea, vomiting and bilateral flank pain. Ultrasound of
the kidneys, ureters and bladder shows loss of normal corticomedullary differentiation with
lack of cortical blood flow. CT shows lack of cortical enhancement on both kidneys with
enhancing medulla. Delayed images show no excretion of contrast into the collecting system.
The most likely diagnosis is? [B5 Q7]

(a) Medullary nephrocalcinosis
(b) Renal cortical necrosis
(c) Papillary necrosis
(d) Acute pyelonephritis
(e) Acute interstitial nephritis

A

Renal cortical necrosis

This is rare cause of acute renal failure with typical imaging features as given. CT is the most
sensitive and specific imaging for this condition. Absent opacification of the cortex with
enhancement of juxtamedullary kidney and poor contrast excretion is diagnostic for this
condition.

107
Q

Regarding hydronephrosis of pregnancy: [B3 Q 34]

A. Left-sided dilated predominates
B. Occurs predominantly due to maternal hormones decreasing ureteric tone
C. Persists at most 3-4 days postpartum in most cases
D. Occurs in 90% of pregnant women by third trimester
E. Usually involves the entire length of the uterus

A

Occurs in 90% of pregnant women by third trimester

In most women dilatation disappears postpartum. Resolution can take between a few days
to several weeks. Maternal hormones play a minor part. Right sided hydronephrosis is more
common

108
Q

A 30-year-old female has continued ipsilateral loin pain following surgery for pelvi-ureteric
junction obstruction. Ultrasound scan shows the renal pelvis to be less dilated than prior to
surgery. MAG-3 renogram is inconclusive about the question of ongoing obstruction, and a
Whitaker test is performed. The maximum pressure difference between the collecting system
(antegrade) needle and the urinary catheter is found to be 10 cmH2O. What should the patient
be advised regarding the findings of the Whitaker test? [B4 Q10]

a. this invasive test is also inconclusive
b. there is no evidence of pelvi-ureteric junction or ureteric obstruction
c. there is ongoing pelvi-ureteric junction obstruction
d. there is a second previously occult level of obstruction
e. the original diagnosis of pelvi-ureteric junction obstruction is in doubt

A

There is no evidence of pelvi-ureteric junction or ureteric obstruction

The Whitaker test is a pressure–flow study to evaluate ureteral obstruction or resistance in
dilated non-refluxing upper tracts. Being invasive and time-consuming, it is usually performed
only when excretory renograms are equivocal. A pressure difference of greater than 15cmH 2 O
is abnormal.

109
Q

An 18-year-old mountain bike enthusiast is suspected of sustaining a renal injury after
attempting a front wheel touch-up manoeuvre. A laceration to the right kidney is noted on CT,
which demonstrates contrast enhancement during the pyelographic phase of the examination.
What is the significance of this finding? [B1 Q58]

A. Pre-existing angiomyolipoma.
B. Active haemorrhage.
C. Devascularization.
D. Renal infarction.
E. Urine leak.

A

Urine leak.

Lacerations generally contain clotted blood and therefore do not enhance on scans obtained
with intravenous contrast. Contrast enhancement during the pyelographic phase of the CT
examination indicates the presence of a urine leak. A delayed scan of 10–15 minutes may
show the extent of the urinary extravasation. Intense enhancement within a laceration during
the early phase indicates active haemorrhage. Focal areas of infarction do not enhance (unlike
contusions). The cortical rim nephrogram is a sign of a devascularized kidney, which occurs
due to laceration of the main renal artery.

110
Q

A 36-year-old labourer working on a building site falls 15 feet from scaffolding on his back
onto a wheelbarrow. He is catheterized in the resuscitation room and is noted to have gross
haematuria. Which of the following CT findings are consistent with a grade 4 renal injury? [B1
Q53]

A. Renal artery avulsion.
B. Shattered kidney.
C. Deep laceration to the collecting system.
D. Subcapsular haematoma.
E. Parenchymal contusion

A

Deep laceration to the collecting system.

Renal injuries are classified into five grades of severity by the
American Association for the Surgery of Trauma (AAST), with
approximately 80% classified as grade 1. Most significant
injuries manifest as haematuria and no significant urinary tract
injury occurs in the absence of gross haematuria and shock. In
addition, most significant renal trauma is associated with
additional visceral injury.

111
Q

A 50-year-old male patient on long-term analgesia presents with a history of macroscopic
haematuria. An intravenous urogram is requested. The preliminary film is unremarkable.
Following intravenous contrast administration, there is a delay in excretion on the right side.
Subsequent images demonstrate a filling defect in the distal ureter with proximal dilatation.
The ureter immediately distal to the filling defect is also mildly dilated, but normal in calibre
further down. What is the diagnosis? [B1 Q3]

A. Non-radioopaque calculus.
B. Transitional cell carcinoma (TCC).
C. Blood clot.
D. Sloughed papilla.
E. Pyeloureteritis cystica.

A

Transitional cell carcinoma (TCC).

The appearance of ureteral dilatation around and below an intraluminal filling defect is
described as the ‘goblet’ sign. This sign indicates that the filling defect is caused by a chronic
process, which allows dilatation of the ureter immediately below to accommodate the lesion.
In addition to TCC, this appearance may rarely be seen with metastatic disease or
endometriosis.

Chronic analgesic use is a risk factor for TCC.

Pyeloureteritis cystica appears as multiple small filling defects in the renal pelvis and ureter,
typically seen in diabetics.

112
Q

A 46-year-old man presents with a 6-week history of painless haematuria. A CT urogram is
performed and this demonstrates a 4-cm left renal mass, which is centrally placed involving
both cortex and medulla. Which of the following findings will be most helpful when
determining if this is a TCC invading the cortex, rather than a renal cell carcinoma invading
the collecting system? [B1 Q64]

A. Presence of calcification.
B. Necrosis within the mass.
C. Renal vein invasion.
D. Maintenance of reniform shape.
E. Hypo-vascularity of lesion to renal parenchyma

A

Maintenance of reniform shape.

Advanced TCC of the collecting system in the kidney can cause distortion of the renal
architecture, but typically the reniform shape of the kidney is maintained. As RCC arises in the
renal cortex, large tumours of this nature typically distort the normal renal outline
Calcification can rarely occur in both RCCs and TCCs, although perhaps more commonly in
RCC. TCC demonstrates calcification in less than 3%, when it is usually diffuse and punctate.

Necrosis can occur in both large RCCs and large TCCs, giving a heterogenous enhancement
pattern on CT. Renal vein invasion is a more typical manifestation of RCC and both these
tumours may be hypo-vascular compared to normal renal parenchyma on a nephrographic
phase of the CT examination.

113
Q

A 58-year-old man presents with haematuria and suprapubic pain. Ultrasound reveals the
presence of an area of bladder wall thickening and a mobile avascular mass within the urinary
bladder. A degree of right hydronephrosis is also demonstrated. Which of the following is true
regarding transitional cell carcinoma (TCC)? [B2 Q52]

a. The ureter is the second commonest site of TCC after the urinary bladder
b. TCC is the most common tumour of the urinary tract
c. In the ureter, the lower ureter is the commonest site
d. It is the commonest tumour arising in the urachus
e. Previous schistosomiasis is a well-recognised risk factor

A

TCC is the most common tumour of the urinary tract

TCC is the commonest cancer in the urinary tract. Whilst a predominant majority of them
develop in the bladder, the renal pelvis is the second commonest site. In the ureter, the upper
ureter is the commonest site. Adenocarcinoma is the commonest tumour in the urachus.
[???] Schistosomiasis is associated with squamous cell carcinoma.

114
Q

Infiltrating papillary TCC is diagnosed from a mass at the renal pelvis. Which is the single best
answer? [B3 Q4]

A. 50% of TCC are infiltrating papillary tumours
B. Have a broad base and frond-like morphology
C. On CT have a density lower than urine and higher than renal parenchyma
D. CT value of TCC is around 50-60 HU
E. Demonstrate avid enhancement post-contrast

A

Have a broad base and frond-like morphology

Over 85% of TCCs are infiltrating papillary tumours. CT density is 8-30 HU, slightly higher than
urine and lower than renal parenchyma. Post-contrast there is mild to moderate
enhancement to 18-55 HU.

115
Q

Which of the following favours a diagnosis of infiltrating papillary TCC rather than ureteral
endometriosis? [B3 Q5]

A. Location in proximal third of ureter
B. Age 35 years
C. An intramural nodule
D. Soft tissue component outside ureter larger than in it
E. High signal intensity on T1 and ‘shading’ on T2

A

Location in proximal third of ureter

Ureteral endometriosis is usually in childbearing age and in the lower third of ureters, often
co-existent with other sites of abdominal and pelvic disease.

116
Q

A patient is found to have a renal pelvis transitional cell carcinoma. The cancer invades
adjacent renal parenchyma and extends into perinephric fat. No significantly enlarged lymph
nodes and no metastases are seen on CT of the chest, abdomen, and pelvis. Which of the
following is the overall stage for this patient’s disease? [B4 Q21]

a. I
b. II
c. III
d. IV
e. V

A

IV

In the TNM staging of urothelial malignancies, T1 refers to invasion of the subepithelial
connective tissue, whereas a T2 tumour invades the muscularis. T3 tumours in the renal pelvis
invade the peri pelvic fat or renal parenchyma, whereas those in the ureter invade the
periureteric fat. T4 tumours invade adjacent organs or perinephric fat, as in this case. For
renal and ureteric transitional cell carcinoma, the group stages I–III are determined by the T
status, all these stages having no involved nodes

117
Q

A patient with a lower ureteric transitional cell carcinoma has an MRI for locoregional staging
purposes and a CT of the abdomen and pelvis for lymph node involvement and metastases. An
8 mm short axis node is recorded. In which of the following abdominopelvic groups would this
be significant by size criteria? [B4 Q41]

a. inguinal
b. common iliac
c. external iliac
d. internal iliac
e. retroperitoneal

A

Internal Iliac

A short axis measurement of 7 mm or greater represents significant enlargement of internal
iliac nodes. Regarding other nodal regions, significant enlargement for inguinal nodes is
10mm, for common iliac 9mm, for external iliac 10mm, for obturator 8mm and for
retroperitoneal nodes between renal arteries and the aortic bifurcation 12mm. In addition to
size, there may be morphological clues to nodal involvement by cancer. Clustering of nodes,
round nodes, nodes with irregular capsules, and nodes sharing CT or MRI characteristics of
the primary tumour (attenuation, signal, cystic or necrotic changes, and contrast-
enhancement pattern) are features suggesting lymph node involvement.

118
Q

A 32-year-old male has been referred from urology for assessment. The patient was involved
in an RTA 2 years ago, during which he sustained a urethral injury. The patient had failed to
attend urology outpatients for follow-up in the interim but has re-attended with recurrent UTIs.
An attempted cystoscopy identified a tight proximal urethral stricture. The urologists have
requested an ultrasound and voiding cystourethrogram (VCUG) to investigate the degree of
bladder outlet obstruction. The ultrasound demonstrates a mild degree of renal pelvic dilatation
bilaterally. The bladder has a residual volume of 500 ml on post-void imaging. The VCUG
reveals reflux into the renal pelvis bilaterally with mild ureteric and pelvic dilatation, but no
calyceal dilatation and preserved forniceal angles. What grade of reflux does this patient have?
[B1 Q6]

A. Grade 1.
B. Grade 2.
C. Grade 3.
D. Grade 4.
E. Grade 5.

A

Grade 3.

Whilst most associated with congenital reflux in children, acquired reflux is also seen, most
commonly due to bladder outlet obstruction or recurrent cystitis.

Vesicoureteric reflux grading:

Grade 1 reflux demonstrates reflux into the ureter.
Grade 2 reflux into the renal pelvis
Grade 3 Mild pelvi-ureteric dilatation.
Grade 4 Moderate pelvi-ureteric dilatation and calyceal blunting, but preserved papillary impressions.
Grade 5 Markedly dilated pelvis and tortuous ureter with obliteration of the forniceal angles and papillary impressions.

119
Q

A 25-year-old female undergoes a hysterosalpingogram (HSG) that reveals a unicornuate
uterus. This is confirmed on MRI, which also demonstrates a non-functioning rudimentary
contra-lateral horn. What further investigation is indicated? [B1 A8]

A. Laparoscopy.
B. Renal imaging.
C. Pelvic ultrasound.
D. Laparotomy and resection of rudimentary horn.
E. Speculum examination.

A

Renal imaging.

Mullerian duct anomalies are associated with renal tract anomalies in up to 30% of cases due
to the close embryologic relationship between the Mullerian and mesonephric ducts. The
most common renal tract abnormality is renal agenesis. Other associated anomalies include
duplicated collecting system, renal duplication, horseshoe kidney, crossed renal ectopy. and
cystic renal dysplasia. Renal imaging is therefore essential in all patients with Mullerian duct
anomalies. Surgical intervention is not required in patients with a unicornuate uterus and a non-
functioning rudimentary horn.

120
Q

A 2-day-old male infant with cryptorchidism and an antenatal diagnosis of dilated bladder and
ureters is referred for a micturating cystourethrogram (MCUG). MCUG reveals a dilated
bladder, tortuous and dilated ureters, dilated posterior urethra, and renal cortical thinning
. Note
is also made of bulging flanks. What is the diagnosis? [B1 Q12]

A. Posterior urethral valve.
B. Congenital mega-cystis and megaureter.
C. Bilateral vesicoureteral reflex.
D. Eagle Barrett syndrome.
E. Meatal stenosis.

A

Eagle Barrett syndrome.

Eagle Barrett syndrome or prune belly syndrome is classically defined as a triad of partial or
complete absence of abdominal musculature (resulting in bulging flanks), cryptorchidism, and
urinary abnormalities as described above. In addition, several respiratory, GI, musculoskeletal,
and cardiovascular anomalies are associated with this syndrome.

It occurs almost exclusively in males. The cause and embryogenesis remain controversial.
Bladder outlet obstruction, mesodermal arrest, and dysgenesis of the yolk sac have been
proposed as possible causes.

121
Q

A 2-day-old male neonate with a right-sided abdominal mass is referred for ultrasound of
abdomen. Ultrasound demonstrates an enlarged right kidney containing multiple non-
communicating cysts of varying size with little normal parenchyma. What is the most common
associated abnormality of the contra-lateral kidney? [B1 Q52]

A. Ectopic ureter.
B. Pelvi-ureteric junction obstruction.
C. Vesico-ureteric reflux.
D. Renal hypoplasia.
E. Renal aplasia.

A

Vesico-ureteric reflux

The abnormality described on ultrasound is multicystic dysplastic kidney (MCDK). It is the
most common form of cystic disease in infants and the second most common cause of an
abdominal mass in a neonate (after hydronephrosis). Obstruction/atresia of the ureter during
the developmental stage is thought to be the etiology.

Bilateral MCDK is uncommon, but associated anomalies of the contra-lateral kidney are seen
in up to 50% of cases. Vesico-ureteric reflux (30–40%) is the most common associated
anomaly, followed by pelvi-ureteric junction obstruction (10–20%).

122
Q

A 37-year-old male undergoes an intravenous urogram and the right ureter is deviated medially
in the lumbar region. Which one of the following could explain this finding? [B2 Q57]
a. Psoas muscle hypertrophy
b. Para-aortic lymphadenopathy
c. Retrocaval ureter
d. Urinoma
e. Abdominal aortic aneurysm

A

Retrocaval ureter aka Circumcaval ureter

Medial deviation of the ureter is seen with retrocaval ureter on the right side and with
retroperitoneal fibrosis. The other conditions listed all cause medial deviation of the ureter in
the lumbar region. Retrocaval ureter is a rare entity which is caused by abnormal
embryogenesis of the IVC. There may be symptoms of right ureteral obstruction and recurrent
urinary tract infections.

123
Q

On an antenatal ultrasound scan, unilateral foetal ureteric dilatation is identified. Follow-up
imaging after birth reveals persistent dilatation of a non-tortuous ureter that is aperistaltic in its
distal segment. There are no other associated urogenital anomalies. Which of the following is
the most appropriate diagnosis? [B4 Q77]

a. prune-belly syndrome
b. primary vesicoureteric reflux
c. primary megaureter
d. neuropathic bladder
e. ureterocele

A

Primary megaureter

The primary megaureter occurs because of developmental aperistalsis of the distal ureter.
The ureter tends to be dilated but straight in a congenital primary megaureter. Primary
vesico-ureteric reflux and obstructive secondary megaureter tend to cause tortuous ureteric
dilatation.

124
Q

An IVU is performed on a teenager. On a 15-minute, full-length radiograph, the right ureter
and collecting system appear normal, while the left collecting system is displaced laterally and
inferiorly, giving a ‘drooping flower’ appearance. What is the most likely congenital anomaly?
[B4 Q79]

a. bilateral ureteral duplication
b. left ureteral duplication
c. left ureteral diverticulum
d. recently passed left ureteric stone
e. crossed fused ectopia

A

Left ureteral duplication

The ‘drooping flower’ appearance on IVU occurs with a duplicated system when the
obstructed dilated upper pole collecting system displaces the contrast-opacified, lower
collecting system laterally and inferiorly. The Weigert–Meyer rule describes the commonest relationship of insertion of the duplicated ureters into the bladder. The upper collecting
system ureter inserts ectopically into the bladder inferior and medial to the orthotopic ureter,
which enters the bladder near the trigone. The ectopic ureter may be stenotic or obstructed
with or without ureterocele

125
Q

A child was diagnosed with prune-belly syndrome. Which of the following is most likely to be
a feature associated with this condition? [B5 Q23]

(a) Polyhydramnios in mother
(b) Usually seen in women
(c) Normal bladder capacity
(d) Vesicoureteral reflux usually present
(e) Normal bladder neck

A

Vesicoureteral reflux usually present

With an abnormal urinary tract, there may be failure of fetal micturition and thus
oligohydramnios. This is seen exclusively in men and reflux is present in most cases. The
bladder neck is typically wide with a tapering dilatation of the posterior urethra.

126
Q

A 10-year-old girl presents with urinary tract infection. Ultrasound and micturating
cystourethrogram demonstrates a left-sided vesicoureteral reflux with reflux to the
pelvicalyceal system without calyceal dilatation or blunting. What is the most likely grade of
the vesicoureteral reflux? [B5 Q45]

(a) Grade I
(b) Grade II
(c) Grade III
(d) Grade IV
(e) Grade V

A

Grade II

Grade I is reflux only in the ureter and not into the pelvicalyceal system.

Grade III is reflux into the pelvicalyceal system with mild dilatation of the ureter and
pelvicalyceal system.

Grade IV and V are more severe dilatations and tortuosity of the ureter and pelvicalyceal
system

127
Q

A 56-year-old female diabetic patient presents to urology with a history of microscopic
haematuria. There is a history of multiple UTIs. The standard urology investigative process in
your hospital consists of ultrasound, intravenous urogram (IVU) and cystoscopy. Ultrasound
is normal. Cystoscopy has found several yellowish raised lesions in the bladder. You are
reviewing the IVU with this information. The IVU shows multiple small filling defects in the
distal left ureter and bladder which are 3–8 mm in size. What is the diagnosis? [B1 Q20]
A. Malakoplakia.
B. Leukoplakia.
C. Pyeloureteritis cystica.
D. Multifocal TCC.
E. Cystitis with distal ureteric reflux.

A

Malakoplakia.

This is an inflammatory response to chronic E. coli infection. It is more common in females
and in diabetic/immunocompromised patients. It most commonly occurs in the bladder and
distal ureter and is multifocal in 75% of cases

128
Q

An 84-year-old diabetic female is investigated for recurrent E. coli urinary tract infections and
microscopic haematuria. An intravenous urogram is performed, which shows numerous small
filling defects in the ureter and small mural plaque-like defects within the bladder.
Which one
of the following is the most likely diagnosis? [B2 Q8]

a. Malakoplakia
b. Leukoplakia
c. Emphysematous cystitis
d. Emphysematous pyelonephritis
e. Pyeloureteritis cystica

A

Malakoplakia

Malakoplakia is the most likely diagnosis based on the history provided. This is a rare granulomatous infection affecting elderly females with a history of E. coli infections. It primarily affects the bladder and affects the remainder of the renal tract with decreased incidence as one progresses proximally.

Leukoplakia may have similar appearance, but is more common in males with bladder
involvement, and is characterised by the passage of gritty soft-tissue flakes.

Pyeloureteritis cystica typically produces multiple round filling defects rather than plaques.

129
Q

An 18-year-old male fractures his pelvis following a motorcycle accident. He is suspected of
sustaining a bladder injury and undergoes CT cystography. This reveals ill-defined contrast
medium within the peri-vesical space with a ‘molar-tooth’ appearance. What is the significance
of this finding? [B1 Q70]

A. Interstitial bladder injury.
B. Intraperitoneal rupture.
C. Extraperitoneal rupture.
D. Combined intra- and extraperitoneal rupture.
E. Bladder contusion.

A

Extraperitoneal rupture.

Over 70% of patients with traumatic bladder injury have a coexisting pelvic fracture. CT
cystography is as accurate as conventional cystography.

Extraperitoneal rupture
* accounts for 80% of all cases of traumatic bladder injury.
* It occurs because of shearing forces or penetrating injury from bony fragments at the base
of the bladder.
* Contrast can also track down into the scrotum or thigh.

Intraperitoneal rupture
* (15% of cases)
* follows a direct blow to a distended bladder, with the tear involving the bladder dome.
Contrast will be seen to outline small bowel loops.

Combined injuries occur in 5%. Interstitial injuries are rare and are detected by contrast
dissecting into the bladder wall. Imaging is frequently normal in the setting of bladder
contusion.

130
Q

A 31-year-old male is involved in a road traffic accident. The patient was catheterised
immediately in A&E and the bladder was found to be empty. A trauma series CT is requested,
and a left-sided pelvic fracture is noted. A CT cystogram is therefore performed and bladder
rupture is diagnosed. Which of the following signs would be an unexpected finding with this
history? [B2 Q20]

a. Contrast extravasation into the paracolic gutters
b. Contrast extravasation into the perivesical fat
c. Contrast extravasation into the anterior abdominal wall
d. Flame-shaped contrast extravasation
e. Contrast extravasation into the upper thigh

A

Contrast extravasation into the paracolic gutters

Extraperitoneal rupture of the bladder is associated with pelvic fractures following trauma
and cystography should be performed if this is suspected. The injury is usually at the base of
the bladder, anterolaterally. Contrast is seen to extravasate with a streaky or flame-shaped
appearance and collects in the space of Retzius, upper thighs, inguinal regions, perivesical fat
and anterior abdominal wall. Contrast in the paracolic gutters suggests intraperitoneal
rupture of the bladder. This is associated with a different method of injury, typically rupture
at the bladder dome following blunt trauma with a distended bladder or secondary to
iatrogenic injury such as cystoscopy.

131
Q

Which of the following indicates T3 disease in a 66-year-old man with bladder cancer? [B3
Q8]

A. Invasion of peri-vesical fat
B. Tumour size 2.5cm
C. Tumour invading inner half or superficial muscle
D. Tumour invading deep muscle
E. Invasion of the rectum

A

Invasion of peri vesical fat

Invasion of peri vesical fat indicates T3 disease. Invasion of surrounding organs, pelvic or
abdominal wall is T4 disease. T1-T2b tumours are treated with a conservative approach
including TURB and local chemotherapy, whereas radical cystectomy and urinary diversion
are reserved for invasive cancer.

132
Q

On CT performed for staging purposes, a primary bladder tumour involves bladder muscle
without peri vesical extension. Malignant enlarged lymph nodes of 4 cm greatest dimension in
the ipsilateral internal iliac and 1.5 cm greatest dimension in the common iliac lymph node
groups are present. Which of the following is the most accurate TNM stage? [B4 Q16]

a. T1 N1 M0
b. T2 N1 M0
c. T2 N2 M0
d. T2 N1 M1
e. T3 N1 M0

A

T2 N2 M0

Bladder cancer achieves the T3 status by peri vesical involvement. The N status is determined
by the greatest dimension of the regional nodes. When the greatest dimension is less than or
equal to 2cm, the nodal status is N1. N2 is for regional nodes measuring 2–5cm. N3 is achieved
when the greatest dimension of the largest regional node is more than 5cm. Inguinal and
retroperitoneal nodes are staged as metastases.

133
Q

An 80-year-old man undergoes cystoscopy for macroscopic haematuria. He is found to have a
6 cm bladder tumour, biopsy of which confirms small-cell bladder cancer. He is considered
suitable for radical treatment. Which of the following is the most appropriate staging strategy?
[B4 Q53]

a. whole-body PET/CT
b. MRI of the bladder
c. MRI of the bladder plus CT of the abdomen and pelvis with intravenous contrast
d. MRI of the bladder plus CT of the chest, abdomen, and pelvis with intravenous contrast
e. MRI of the bladder plus CT of the brain, chest, abdomen, and pelvis with intravenous
contrast

A

MRI of the bladder plus CT of the chest, abdomen, and pelvis with intravenous contrast

MRI is indicated for local (in fact locoregional) staging. CT of the chest is required in addition
to the abdomen and pelvis because of the histological tumour type. 18 FDG PET is not useful
for staging urothelial tumours because of the urinary excretion of this radiotracer.

134
Q

A 40-year-old diabetic presents with Urinary Tract Infection (UTI). Abdominal radiograph
demonstrates small rounded curvilinear lucencies outlining the bladder wall. What is the
diagnosis? [B3 Q48]

A. Emphysematous cystitis
B. Enterovesical fistula
C. Penetrating trauma
D. Post-cystoscopy
E. Pneumatosis intestinalis

A

Emphysematous cystitis

Emphysematous cystitis occurs more commonly in females and is usually due to E.coli.
Gas is present in the bladder mucosal lumen. On ultrasound there is a thickened bladder wall
with echogenic foci and acoustic shadowing

135
Q

A 55-year-old female patient presents to your hospital with a history of recurrent UTIs and
gross haematuria. Repeated urine cultures are negative, but analysis reveals copious white and
red cells in the urine. The patient fails to improve with antibiotics.

A CT scan of renal tracts is carried out, which shows an atrophic right kidney containing
calcification. There is also an area of increased density on the unenhanced portion of the scan
noted in the upper pole of the kidney, with overlying cortical thinning. There are multiple
strictures noted in the ureter, with intervening areas of dilatation, giving a corkscrew
appearance. There is extensive coarse calcification noted in the wall of the bladder.

A CXR is carried out and is normal. Early morning urine collections finally identify
mycobacterium tuberculosis in the urine, confirming the suspicion of renal and urinary tract
TB. Which of these features is atypical of renal TB? [B1 Q55]

A. High-density material in the calyceal system.
B. Bladder calcification.
C. Renal calcification.
D. Normal CXR.
E. Corkscrew appearance to the ureter.

A

Bladder calcification

Whilst this can be seen in TB, it is more typically associated with schistosomiasis. TB usually
causes scarring and a reduced capacity bladder. Renal calcification is typical. The areas of
increased attenuation within the calyceal system represent areas of coalescing caseating
granulomas and may have associated calcification. Scarring can also cause stenosis of calyces,
causing focal obstruction. Occasionally a small calcified kidney is found, evidence of auto-
nephrectomy. Passage of the infection via the urine into the ureters causes focal stenoses,
which can coalesce to cause a long stricture, or give a beaded or corkscrew appearance to the
ureter. Whilst renal TB results from spread from a primary pulmonary infection, the CXR is
only abnormal in 25–50% of cases and is therefore not helpful.

136
Q

A 49-year-old African male presents to the outpatient urology clinic with a five-month history
of macroscopic haematuria. A plain KUB X-ray is requested, which reveals thin arcuate
calcification outlining the bladder and the distal ureters. Which one of the following causes is
most likely? [B2 Q43]

a. Transitional cell carcinoma
b. Squamous cell carcinoma
c. Schistosomiasis
d. E. coli cystitis
e. Proteus cystitis

A

Schistosomiasis

The differential for bladder calcification includes tuberculosis, post-radiotherapy cystitis, urachal carcinoma, TCC, and squamous cell carcinoma. However, schistosomiasis is the commonest cause, especially in the African population, where it is often endemic. The bladder is usually a normal size and shape, with thin curvilinear calcifications. Ureteric strictures, inflammatory pseudo-polyps and vesico-ureteric reflux are seen in addition to bladder and ureteric calcification.

137
Q

A CT KUB is performed on a 55-year-old South African man with unilateral loin pain. This
demonstrates moderate ipsilateral hydroureteronephrosis with a stricture in the distal ureter.
There is also widespread bladder calcification and bilateral distal ureter calcification. The
responsible organism is most likely to be which of the following? [B4 Q48]

a. Escherichia coli
b. Schistosoma mansoni
c. Schistosoma haematobium
d. Schistosoma japonicum
e. Mycobacterium tuberculosis

A

Schistosoma haematobium

Schistosomiasis (bilharzia) is a parasitic infection that, worldwide, is the commonest cause of
bladder wall calcification. Schistosoma japonicum and S. mansoni cause gastrointestinal tract
infection, while S. haematobium affects the genitourinary tract.

Schistosomiasis is endemic in South Africa, Egypt, Nigeria, Tanzania, Zimbabwe and Puerto
Rico. The calcification spreads proximally up the ureters. In contrast, tuberculosis begins in
the kidneys and spreads distally. Transitional cell carcinoma and cyclophosphamide-induced
cystitis also cause bladder wall calcification. Causes of calcification within the urinary bladder
lumen include stones and encrusted foreign bodies such as catheter balloons.

138
Q

A 60-year-old female with urinary retention and pelvic pain is investigated with MRI of the
pelvis. On sagittal T2W images, the bladder is seen to be 2 cm below the pubococcygeal line.
What is the likely cause of the patient’s symptoms? [B4 Q90]

a. anterior rectocele
b. enterocele
c. cystocele
d. rectal prolapse
e. bladder intussusception

A

Cystocele

Suspected pelvic floor prolapse can be investigated with MRI. Usually, there is minimal
movement of pelvic organs even on maximal strain. When the pelvic floor is lax, the organs
descend below the pubococcygeal line by 1–2cm. When descent exceeds 2cm, the prolapse
may require surgical intervention. An enterocele describes small bowel descending 2cm or
more between vagina and rectum. Anterior bulging of the rectal wall is known as a rectocele,
while bladder descent of more than 1cm is a cystocele

139
Q

A 45-year-old migrant from South Africa with history of recently treated oesophageal varices
presents with haematuria. Plain radiograph of abdomen and pelvis shows curvilinear calcification in the wall of urinary bladder. There is bilateral hydroureters and hydronephrosis.
What is the most likely diagnosis? [B5 Q19]

(a) Bladder carcinoma
(b) Portal hypertension
(c) Tuberculosis of urinary bladder
(d) Schistosomiasis
(e) Rhabdomyosarcoma of bladder

A

Schistosomiasis

Schistosomiasis is endemic in southern and east Africa and is a result of infection by
Schistosoma haematobium. Ova are laid into the submucosa of the lower urinary tract,
causing an extensive fibrosing with later calcification. Ova migrating into the portal venous
system result in a fibrosing granulomatous reaction leading to portal hypertension and
oesophageal varices.

140
Q

A 22-year-old male front seat passenger is admitted following a RTA. On examination in A&E,
blood is noted at the external urethral meatus and there is swelling of the penis. An urethrogram
is performed which demonstrates contrast extravasation below the urogenital diaphragm only.
What type of urethral injury does this represent? [B1 Q63]

A. Anterior urethral injury.
B. Disruption of the membranous urethra.
C. Bladder neck injury extending into the proximal urethra.
D. Bladder base injury.
E. Penile fracture.

A

Anterior urethral injury.

Urethral injuries are rarely life-threatening but have significant long-term morbidity.
Complications include stricture, incontinence, and impotence. The male urethra extends from
the bladder base to the external meatus and is divided into the posterior (prostatic and
membranous) and anterior (bulbous and penile) urethra. The anterior and posterior urethra
are separated by the urogenital diaphragm. The Goldman classification of urethral injury
emphasizes anatomic location.

141
Q

A 36-year-old man suffers pelvic fracture following a road traffic accident. On examination,
blood is noted at the urethral meatus and the patient has urinary retention. Regarding urothelial
injuries: [B2 Q16]

a. Associated bladder injuries are seen in 50% of patients
b. Anterior urethral injuries are commoner with pelvic fractures
c. They are more commonly associated with pelvic fractures in females rather than males
d. Posterior urethral injuries can be seen in up to 20% of pelvic fractures in males
e. Impotence is a rare complication of male urethral injury

A

Posterior urethral injuries can be seen in up to 20% of pelvic fractures in males

Urethral injuries are seen in up to 20% of male patients following pelvic fractures. They are
much less common in women. The posterior urethra is the commonest site; impotence can
develop in up to 40% of these patients.

142
Q

A 25-year-old male driver was admitted to the Accident & Emergency Department after a road
traffic accident. Plain radiography shows a fractured pelvis, and the patient is unable to pass
urine. The registrar notes blood at the urethral meatus. What is the appropriate management
for this condition? [B5 Q49]

(a) Foley’s catheter insertion to drain urine
(b) Retrograde urethrogram to exclude urethral injury
(c) Micturating cystourethrogram
(d) Cystography
(e) Antegrade urethrography

A

Retrograde urethrogram to exclude urethral injury

In patients with pelvic fractures, bloody meatus, and inability to void should raise the
possibility of urethral injury. A retrograde urethrogram should be performed to exclude
urethral injury before inserting a Foley’s catheter or cystography for bladder ruptures.

143
Q

Cystoscopy is attempted on a 65-year-old female for persistent microscopic haematuria, but
the scope cannot be advanced along the urethra. A biopsy is taken, and MRI is performed.
Axial T2W images show a mass of high signal intensity disrupting the normal, target-like,
zonal anatomy of the urethra. Which of the following cell types is the most likely histology
from the biopsy? [B4 Q57]

a. squamous cell
b. transitional cell
c. adenocarcinoma
d. clear cell
e. mastocyte

A

Squamous cell

Urethral tumour is rare and occurs more in women than in men. Squamous cell carcinoma is
the most common histological type followed by transitional cell carcinoma and then
adenocarcinoma. MRI is the technique of choice for local staging

144
Q

A 65-year-old man undergoes a penile MR for staging of penile cancer. Which of the following
is true? [B2 Q5]

a. Corpus spongiosum has a high signal on T1-weighted images
b. On T2-weighted images, the periurethral tissue has high signal intensity relative to the corpus
spongiosum
c. Corpus spongiosum enhances more rapidly following gadolinium as compared to the corpora
cavernosa
d. MR can reliably differentiate between Buck’s fascia and tunica albuginea
e. A pelvic coil is preferred for local staging of penile cancers

A

Corpus spongiosum enhances more rapidly following gadolinium as compared to the corpora
cavernosa

Both corpus spongiosum and the corpora cavernosa are low on T1, High on T2.
The periurethral tissue is low signal on T2
MR cannot reliably differentiate between Buck’s fascia and tunica albuginea. They are depicted as a single, thick, low-signal rim.
A surface coil is used for local disease staging.

145
Q

A 50-year-old man complains to his general practitioner of painful sexual intercourse. An
ultrasound examination of the penis is performed, which identifies dense, shadow-casting
abnormalities of the periphery of both corpora cavernosa. What is the likely diagnosis? [B4
Q22]

a. priapism
b. Zoon’s balanitis
c. penile squamous cell carcinoma
d. Peyronie’s disease
e. balanitis xerotica obliterans

A

Peyronie’s disease

The cause of Peyronie’s disease is unknown, but the result is fibrous thickening of Buck’s
fascia and the septum between the corpora cavernosa. Calcified plaques are also seen. The
fibrous areas do not engorge with blood, causing the penis at erection to be bent; this can
make intercourse painful or impossible.

  • Priapism is persistent painful erection of the penis but is not associated with penile
    calcification.
  • Zoon’s balanitis is an idiopathic lymphocytic inflammatory condition of the penis, which
    may respond to topical steroid but is also treated by circumcision.
  • Balanitis xerotica obliterans is the severe form of penile lichen sclerosus, which is an
    uncommon inflammatory dermatosis. It can cause phimosis and urinary retention.
146
Q

On an axial MRI of the penis, three tubular masses of tissue, occupying much of the cross-
sectional area of the penis, are surrounded by a T1- and T2-hypointense layer. Breach of this
layer upstages a penile cancer from T1 to T2. From the following choices, name this structure
or structures. [B4 Q24]

a. corpora cavernosa
b. corpora spongiosa
c. cavernosal arteries
d. urethra
e. tunica albuginea

A

Tunica albuginea

Invasion of the corpora cavernosa or corpora spongiosa by a penile cancer is via the tunica
albuginea, making the local stage T2. If the urethra, found centrally within the corpora
spongiosa, is involved, it becomes T3. Each corpus cavernosum contains a central artery.
Having rapidly flowing blood, this will also be hypointense on both T1W and T2W images
because of flow void.

147
Q

A 55-year-old man has biopsy-proven penis cancer. An MRI is performed. Which of the
following is the best reason for performing this examination? [B4 Q39]

a. to confirm the diagnosis
b. to refute the diagnosis
c. to perform local staging
d. to assess metastatic spread
e. to assess regional lymph node involvement

A

To perform local staging

MRI offers good soft-tissue contrast that is of value in local staging of the primary tumour.
Local extent is used to guide the type of treatment, which includes partial penile amputation,
total penectomy and radiation therapy. MRI will also show enlarged regional lymph nodes,
but the principal purpose of MRI is local staging. CT can also be used for nodal spread and
metastatic disease.

148
Q

A 35-year-old male is prompted to see his general practitioner by his wife, who has noticed
blood in the man’s semen. An ultrasound scan of the scrotum is performed. What is the most
common appearance that would accompany this symptom? [B4 Q46]

a. normal appearances
b. enlarged spermatic cord, epididymis and testicle with decreased echogenicity
c. testicular enlargement showing a hypoechoic lesion with a fluid–fluid level
d. paratesticular ‘bag of worms’ appearance
e. scrotal skin thickening

A

Normal appearance

Investigation in most cases of haemospermia is not fruitful. In patients under 40 years, the
causes are usually idiopathic or inflammatory (prostatitis, epididymo-orchitis, urethritis and
urethral warts). The same causes apply in those over 40 years, but further possible causes
include prostate cancer, benign prostatic hypertrophy, prostatic or seminal vesicle calculi,
hypertension and carcinoma of the seminal vesicles

149
Q

On MRI of the penis, a squamous cell carcinoma is typically seen as hypointense to the corpora
on T1W and T2W images. What is the most likely appearance of this cancer on T1W images
following intravenous contrast administration? [B4 Q88]

a. no change in signal compared with that on the pre-contrast T1W sequence
b. post-contrast enhancement greater than that of the corpus spongiosum but less than that of
the corpora cavernosa
c. post-contrast enhancement less than that of all corpora
d. post-contrast enhancement greater than that of all corpora
e. post-contrast enhancement greater than that of the corpora cavernosa but less than that of the
corpus spongiosum

A

Post-contrast enhancement less than that of all corpora

Most cancers of the penis (95%) are squamous cell carcinoma, but basal cell carcinoma,
sarcoma, melanoma, lymphoma and urethral transitional cell carcinoma are also possible.
Typical appearances of a primary penile cancer are of an ill-defined infiltrating lesion
hypointense to the corpora on both T1W and T2W images. Tumours enhance following
contrast, but to a lesser degree than the normal corporal bodies. Signal characteristics of
melanotic melanoma will be notably different from the other tumour types, returning a bright
signal on T1W images.

150
Q

A 73-year-old male patient is referred from the urologists with a new diagnosis of prostate
cancer. He is having an MRI scan to help stage the tumour. The patient reports that he has had
a trans-rectal ultrasound (TRUS) biopsy done but cannot remember when and the date is not
on the form. The following information is present on the request form: ‘The tumour is not
palpable on digital rectal examination (DRE). The prostate specific antigen (PSA) is 2.4. One
of four biopsies was positive and the Gleason score from this was 5.’ When you do the MRI
scan there is low signal in the peripheral zone on the left. There is also low signal on the right,
which extends into the seminal vesicles. The images of the pelvis and para-aortic region do not
show any evidence of lymphadenopathy. Which of these options would you choose for your
report? [B1 Q39]

A. Repeat scan in 4 weeks.
B. T1 disease.
C. T2a disease.
D. T2c disease.
E. T3b disease.

A

Repeat scan in 4 weeks.

There are a number of important points in this question. Firstly, the date from biopsy must
be known to avoid misinterpreting haemorrhage as disease on MRI. Fortunately, the
urologists have provided you with the salient information necessary to estimate disease stage
from the Partin’s tables. Whilst these are clinical features, they are essential to know when
interpreting MRI. The features described indicate a <1% risk of this patient having
extracapsular disease. In combination with the unknown date of the biopsy, it would be
prudent to exclude the possibility that this low signal in the seminal vesicles is not
haemorrhage by repeating the scan. If this does represent disease invasion of the seminal
vesicles, then it would indicate T3b disease.

151
Q

A 56-year-old male patient has presented to the urologists with a PSA of 17 ng/ml. He does
not have a mass palpable on DRE. Two separate TRUS investigations, with a total of eight
biopsies, have failed to yield a tissue diagnosis. The urologists have asked you to carry out an
MRI to help guide their future biopsies. As is routine in these difficult cases, magnetic
resonance spectroscopy (MRS) is carried out on suspicious areas to provide extra information.
The MRI identifies an area of low signal in the anterior peripheral zone. This reveals an
elevated choline and creatine peak, and a reduced citrate peak. Which of these features is
suggestive of carcinoma? [B1 Q45]

A. Elevated choline.
B. Reduced polyamine peak.
C. Reduced citrate peak.
D. None of them, MRS is only sensitive in the transition zone.
E. All of them.

A

All of them.

MRI is not a primary investigation in the diagnosis of prostate cancer, but it is used in staging
known disease. Occasionally, in a patient with a high risk of prostate carcinoma, as in this case,
when the urologists have repeatedly failed biopsies, MRI can be used to help guide biopsy. In
this setting MRS can also be used to increase confidence in the diagnosis. As with all prostate
imaging for cancer, the results are more reliable in the peripheral zone due to the variability
of appearances in the transitional zone. Choline is elevated in prostate cancer and is thought
to reflect increased cell membrane turnover. Creatine, also detected on MRS, is unchanged.
Polyamine is reduced. Citrate, which is stored in normal prostatic cells, is reduced,
presumably because of reduced normal function within cancerous cells. The (creatine +
choline)/citrate ratio has been used to help discriminate prostate cancer from normal
prostate. The role of MRS in the transitional zone is unclear.

152
Q

A 69-year-old man undergoes an MR for staging of prostate cancer. Which of the following is
true regarding MR imaging of the prostate gland? [B2 Q6]

a. The zonal anatomy is best depicted on T1-weighted images
b. The central zone has a higher signal than the peripheral zone on T2-weighted images
c. The low signal intensity posterolateral to the capsule on T2-weighted imaging represents the
seminal vesicles
d. The proximal urethra is usually identified easily
e. post-contrast, the peripheral zone enhances more than the central zone

A

Post-contrast, the peripheral zone enhances more than the central zone

The zonal anatomy is best depicted on T2-weighted images. The proximal urethra is not
routinely identifiable unless the patient is catheterised or has had previous TURP. Seminal
vesicles are bright on T2-weighted images; the low-intensity structures indicate the
neurovascular bundles. The peripheral zone has a higher signal on T2-weighted images and
enhances more.

153
Q

A 70-year-old man undergoes an MR examination of the prostate to assess the stage of prostatic
carcinoma. Which of the following is the least accurate? [B2 Q17]

a. Obliteration of the recto-prostatic angle is suggestive of extracapsular spread
b. Bladder and rectal involvement are best seen on coronal images
c. Focal low signal in the seminal vesicles on T2-weighted imaging is a feature of invasion
d. On T2-weighted images, prostate cancer usually demonstrates low signal intensity in
contrast to the normal peripheral zone
e. Prostatic volume measurements are bigger on CT than MR

B

A

Bladder and rectal involvement are best seen on coronal images

Bladder and rectal involvement are best appreciated on axial and coronal images. MR is much
more accurate for prostatic volume assessment and CT usually overestimates prostatic
volume.

154
Q

A 60-year-old man with elevated PSA and a Gleason 4+3 tumour undergoes investigation with
MRI of the prostate. Which is the single best answer? [B3 Q7]

A. Tumour of the gland is low signal on T2
B. Tumour appears bright on the Apparent Diffusion Coefficient (ADC)
C. Tumour appears dark on high b-value DWI images
D. Seminal vesicle involvement indicates T2 disease
E. The inguinal nodes are the first to enlarge with disease progression

A

Tumour of the gland is low signal on T2

Tumour appears as a focus of low signal compared to the normal high signal peripheral zone
on T2. T3 disease indicates extracapsular extension and/or seminal vesicle involvement.
Tumour appears dark on ADC and bright on high b-value DWI images. Obturator nodes are
first to become involved.

155
Q

In prostate cancer, which best describes changes in Magnetic Resonance Spectroscopy (MRS)?
[B3 Q22]

A. Elevated choline
B. Reduced citrate
C. Elevated choline:citrate ratio
D. Elevated citrate:choline ratio
E. Combining MRI with MRS has no increased accuracy in diagnosis of prostate cancer

A

Elevated choline: citrate ratio

Elevated choline + creatine: citrate ratio best describes MRS features of prostate cancer.

156
Q

Regarding MRI prostate seminal vesicles: [B3 Q49]

A. Usually low on T2
B. Usually high SI on Diffusion Weighted Imaging (DWI)
C. Typically have low ADC values
D. Are high signal on T2 when atrophic
E. Are low signal when invaded with tumour

A

Are low signal when invaded with tumour

Seminal vesicles may appear low in signal on T2 when atrophic, empty or with tumour
involvement.

157
Q

Regarding prostate MR: [B3 Q50]

A. The Peripheral Zone (PZ) and Transitional Zone (TZ) are difficult to distinguish on MR
B. Volume of prostate gland > 60cc precludes brachytherapy
C. The normal PZ is typically heterogeneous on T2
D. Haemorrhage appears as low SI on T1
E. Tumours occur more commonly in TZ than PZ

A

Volume of prostate gland > 60 cc precludes brachytherapy

Contraindications to brachytherapy include a volume of over 60cc.

158
Q

Zonal anatomy of the prostate is best seen in which of the following sequences? [B5 Q48]

(a) T1-weighted images
(b) T2-weighted images
(c) Proton density
(d) STIR
(e) T1 fat saturation

A

T2 weighted images

These demonstrate the zonal anatomy of the prostate well. The prostatic urethra serves as a
reference point. The peripheral zone returns high signal compared with the central or
transitional zones.

159
Q

A patient with urinary symptoms and raised PSA was diagnosed with carcinoma of the prostate
on histopathology. Which of the following appearances is suggestive of carcinoma of the
prostate on MRI? [B5 Q1]

(a) Peripheral zone lesion with high signal on T1
(b) Peripheral zone lesion with low signal on T1
(c) Peripheral zone lesion with high signal on T2
(d) Peripheral zone lesion with low signal on T2
(e) Peripheral zone lesion with low signal proton density

A

Peripheral zone lesion with low signal on T2

Prostatic carcinoma is best seen as an area of low signal intensity in the peripheral zone of
prostate on T2. The normal glandular tissue returns high signal on T2 images.

160
Q

Which of the following most favours a diagnosis of adult prostate sarcoma rather than
adenocarcinoma? [B3 Q19]

A. Age of patient: 50
B. Size of tumour: 2.3cm
C. Absence of calcification
D. Solid and cystic components
E. Invasion of the bladder wall

A

Solid and Cystic Components

Prostatic sarcomas represent 0.1% of all primary prostate malignancy in adults. Mean age is
younger than that of adenocarcinoma. Typically presents as a large solid and cystic mass with
rapid hypervascular and heterogenous soft tissue occupying all or most of the prostate

161
Q

A 70-year-old man with biopsy-proven prostate carcinoma has an MRI of the prostate to assess
suitability for radical prostatectomy. There is bilateral, multifocal, peripheral-zone, low-signal
change on small FOV T2W images in locations corresponding on more than one scan plane.
These changes are confined within the prostatic capsule. However, enlarged lymph nodes are
seen in the left internal and common iliac groups. No other disease is seen, including on a CT
of the abdomen and pelvis and a radioisotope bone scan. What is the TNM stage of this
patient’s cancer? [B4 Q36]

a. T2b N1 M0
b. T2c N1 M0
c. T2c N1 M1
d. T3 N1 M0
e. T3 N1 M1

A

**T2c N1 M1 **

Being bilateral prostate-confined disease, this cancer is T2c.
Internal iliac, obturator, external iliac and sacral are the regional nodal groups.
Common iliac, para-aortic and inguinal involved lymph nodes are regarded as metastases.

T1 – Not radiologically visible,
T2a – less than half of one side
T2b – more than half of one side
T2c – bilateral,
T3a – breaches the prostatic capsule
T3b – invades seminal vesicles,
T4 – other pelvic tissue.

162
Q

A patient with a raised PSA has MRI of the prostate. There is diffuse, low-signal change
throughout the peripheral zone on T2W images. At the right base, there is focal high signal on
T1W images. At the left base there is restricted diffusion. In the right mid-gland, spectroscopic
analysis reveals a choline plus creatine/ citrate ratio of considerably less than 0.8. In the left
mid-gland, there is a diminished contrast wash-in rate. At the apex, the relative peak
enhancement is less than in other regions of the prostate. Which is the most likely site of focal
prostate carcinoma? [B5 Q62]

a. left base
b. right base
c. left mid-gland
d. right mid-gland
e. apex

A

Left Base

In staging prostate cancer with MRI, the large-FOV T1W sequence is useful for demonstrating
haemorrhage (high signal), enlarged lymph nodes and bone metastases. Small-FOV T2W
images of the prostate in axial, coronal and sagittal planes show the zonal anatomy well, with
normal peripheral zone returning high signal. Cancer within the peripheral zone typically
returns low signal on T2W images. Other typical findings in carcinoma of the prostate are
increased relative peak enhancement, increased contrast wash-in rate, reversal of the choline
plus creatine/citrate ratio on spectroscopy, restricted diffusion and increase in permeability
on pharmacokinetic modelling.

163
Q

Into which of the following lymph node groups does lymph from the scrotum initially drain?
[B4 Q49]

a. para-aortic at the L1–2 level
b. superficial inguinal
c. obturator
d. internal iliac
e. presacral

A

Scrotum - Superficial Inguinal

Penile body - Superficial Inguinal
Proximal penis - Deep Inguinal
Testes - Para-aortic nodes at the L1–2 level.

Appreciation of these patterns of lymph drainage is of vital importance when staging
testicular, penile, and other scrotal malignancies

164
Q

A 34-year-old man presents with a dull ache and a focal non-tender lesion in the right inguinal
region. It is heterogeneous on ultrasound and CT. On MR, it has a heterogeneous signal
intensity on T1 - and T2 - weighted imaging, which enhances post-gadolinium. Which of the
following is the likely diagnosis? [B2 Q4]

a. Haematoma
b. Lipoma of the cord
c. Neurofibroma
d. Abscess
e. Malignancy in an undescended testis

A

Malignancy in an undescended testis

Lipoma of the cord will have a high signal on both T1-weighted and T2-weighted images.
Neurofibroma will demonstrate a target sign on T2-weighted images and is of low attenuation
on CT. Abscess will be clinically apparent, hypoechoic on ultrasound and have high signal on
T2-weighted images. Haematomas are usually of higher attenuation on CT with varying
appearances on MR, but do not demonstrate contrast enhancement.

165
Q

Which of the following favour chronic rather than acute/ subacute torsion of the spermatic cord?
[B3 Q2]

A. Enlarged testis with spherical morphology
B. Good identification of the lobular architecture of the affected testis
C. Thickened mediastinum, less echogenic than contralateral
D. Enlarged and more spherical epididymis
E. Hypoechoic small and harder testis

A

Hypoechoic small and harder testis

Absence of hydrocele and normal scrotal wall/skin are other features of chronic torsion.

166
Q

A 72-year-old man presents with a palpable mass adjacent to the right testis. An ultrasound
scan is performed, and this demonstrates a large epididymal cyst. In addition, there is an
abnormal area within the posterolateral aspect of the right testis, which is rounded in shape on
transverse scanning, but more elongated on longitudinal scanning. It is hypoechoic, with
multiple small cystic areas giving a ‘sponge-like’ consistency. There is no flow within this on
colour Doppler imaging. What is the most likely cause of this? [B1 Q23]

A. Thrombosed intra-testicular varicocele.
B. Cystic degeneration in a testicular infarct.
C. Chronic testicular abscess.
D. Tubular ectasia of the rete testis.
E. Epidermoid cyst.

A

Tubular ectasia of the rete testis.

  • Also known as: Cystic transformation of the rete testis
  • Results from: Partial or complete obliteration of the efferent tubules → Ectasia →
    Cysts
  • Frequently bilateral but may be asymmetric.
  • The site: In or adjacent to the mediastinum testis
  • Associated with: Epididymal cysts or spermatoceles.
  • The key: The elongated shape that replaces the mediastinum.

Intratesticular varicoceles can occur, but they are very rare. It would be even rarer for it to be
thrombosed.

Testicular abscess would not usually be multi-cystic but would be more rounded with an
irregular wall and low-level internal echoes.

Testicular infarction usually manifests as a hypoechoic mass that is largely avascular. It is not
usually cystic.

Epidermoid cyst is a rare benign tumour of germ cell origin. It is usually rounded or oval and
classically has a target or ‘onion skin’ appearance of alternating layers of hyper- and hypo-
echogenicity. The outer wall is typically hyperechoic and sometimes calcified.

167
Q

Considering solid lesions of the para-testicular space, which is the most common benign
tumour? [B3 Q20]

A. Adeno-carcinoid tumour
B. Lipoma
C. Haemangioma
D. Leiomyomas
E. Papillary cystadenomas

A

A

**adeno-carcinoid tumour **

30% of all para-testicular masses and most common benign neoplasm. Hamartomatous
lesions are of probable mesothelial origin in young adults and usually present either as a
painless mass or an incidental finding.

168
Q

A 16-year-old male with a history of recurrent sudden severe testicular pain at night has a
surgical scrotal exploration, which establishes that one testicle has a high insertion of the tunica
vaginalis on the spermatic cord. What proportion of patients with this condition will have
bilateral disease? [B4 Q23]

a. none
b. 10%
c. 25%
d. 65%
e. all

A

65%

A bell-clapper deformity is described. This is bilateral in 50–80% of cases. High insertion of
the tunica vaginalis on the spermatic cord means that this tunic completely surrounds the
testis, epididymis and distal spermatic cord, allowing intravaginal torsion. Extravaginal torsion
is rare and occurs when the testis and tunic twist at the external ring.

169
Q

A 25-year-old previously well man presents with non-specific scrotal pain. Ultrasound shows
numerous bilateral hyperechoic shadows in the testes measuring 1–2 mm. There is no acoustic
shadowing seen. The most likely diagnosis is? [B5 Q3]

(a) Post-inflammatory changes
(b) Haemorrhage with infarction
(c) Testicular scarring
(d) Testicular microlithiasis
(e) Large cell calcifying Sertoli cell tumour

A

Testicular microlithiasis

This occurs when there is a defect in the phagocytic activity of Sertoli cells leaving
degenerated intratubular debris behind. This condition is usually asymptomatic and
ultrasound appearances are typically as described. When testicular microlithiasis is
discovered, regular follow up should be performed due to risk of developing a testicular
neoplasm.

170
Q

An 80-year-old man presented with bilateral testicular lumps. Ultrasound of the testis shows
small, septated, cystic lesions in the mediastinum testis, the right worse than the left. These
lesions are avascular. On MRI, the lesions return low signal on T1 while they are isointense to
testis on T2. What is the most likely diagnosis? [B5 Q22]

(a) Teratoma
(b) Tubular ectasias of rete testis
(c) Epidermoid cyst
(d) Spermatocele
(e) Varicocele

A

Tubular ectasias of rete testis

This is usually seen in older men and is thought to be secondary to cystic dilatation of the rete
testis. This is a benign condition and an important differential is teratoma. Ultrasound
appearances and MRI features are characteristic. These lesions are isointense (sometimes
undetectable) on T2 (unlike teratoma).

171
Q

A 65-year-old diabetic in shock is brought to the Accident & Emergency Department after
collapse at home. He has a 6-day history of progressive scrotal swelling and pain. Ultrasound
shows scrotal thickening and extensive echogenic shadows in the subcutaneous layer with
posterior acoustic shadowing suggesting air. What is the most likely diagnosis? [B5 Q29]

(a) Acute epididymo-orchitis
(b) Fournier’s gangrene
(c) Hernia
(d) Normal variant
(e) Traumatic

A

Fournier’s gangrene

Fournier’s gangrene is a progressive necrotising fasciitis in men. Thickening of the scrotal skin
and air in the subcutaneous layer are diagnostic

172
Q

A 40-year-old Caucasian man presented with a painless left testicular nodule. Ultrasound
shows a well-circumscribed, encapsulated, avascular and round lesion measuring 4 cm in size
in the left testis. It shows an ‘onion-ring’ appearance of alternating areas of hypo- and
hyperechogenicity. On MRI, the lesion shows high signal on T1 and T2 sequences. What is the
most likely diagnosis? [B5 Q46]

(a) Seminoma
(b) Teratoma
(c) Torsion testis
(d) Epidermoid cyst
(e) Lymphoma of testis

A

Epidermoid cyst

These are the typical radiological appearances of an epidermoid cyst of testis. The ‘onion-ring’
appearance is secondary to alternating layers of compacted keratin and desquamated
squamous cells. The water and lipid contents of the cyst result in high signal on both T1 and
T2.

173
Q

A 72-year-old man presents with a 4-month history of painless enlargement of the right testis.
He undergoes scrotal ultrasound, which demonstrates a uniformly hypoechoic lesion that is
enlarging and almost replacing all the normal parenchyma of the right testis. It has also caused
enlargement of the right epididymis and adjacent spermatic cord. Review of the left testis
shows a 2.5-cm hypoechoic lesion with normal spermatic cord and epididymis. What is the
most likely diagnosis? [B1 Q19]

A. Lymphoma.
B. Leukaemia.
C. Testicular metastases.
D. Bilateral seminoma.
E. Granulomatous epididymo-orchitis.

A

Lymphoma.

Clinically testicular lymphoma is distinct from other testicular neoplasms in that it occurs in a
much older age group. It is the most common testicular neoplasm over the age of 60 years. It
is also the most common bilateral testicular neoplasm (up to 38% of cases). The epididymis
and spermatic cord are commonly involved. The sonographic appearance is variable and
indistinguishable from germ cell tumours. Typically, they are discrete hypoechoic lesions that
may completely infiltrate the testicle.

Primary leukaemia of the testis is rare, although it is a common site of leukaemia recurrence
in children. The sonographic appearances are very variable, as the tumour may be unilateral
or bilateral, focal or diffuse, hypoechoic or hyperechoic.

Testicular metastases, other than those from lymphoma or leukaemia, are very rare. Primary
sites reported include prostate and lung. They are generally seen in the setting of widespread
disease and are rarely the presenting complaint.

Seminoma is the most common germ cell tumour. The average age of presentation is
approximately 40 years. It is typically uniformly hypoechoic on ultrasound and they are only
rarely bilateral (2%).

Granulomatous orchitis may manifest as a testicular mass, but this would be very unusual.
Typically this process tends to involve the epididymis first and to a much greater degree than
the testis. Pathogens include TB, syphilis, fungi, and parasites

174
Q

A 38-year-old man with a swollen right hemi-scrotum has an ultrasound examination. Which
of the following is true? [B2 Q30]

a. The epididymis is hypoechoic compared to the normal testis
b. Seminomas are most commonly hyperechoic compared to the normal testis
c. The majority of extra-testicular tumours are benign
d. Lipomas are the commonest intra-testicular benign tumours
e. Epidermoid cysts are most seen in the head of the epididymis

A

The majority of extra-testicular tumours are benign

The epididymis is iso- or hyperechoic compared to the testis. Seminomas are homogenous
masses and hypoechoic to the testis. Epidermoid cysts are the commonest intra-testicular
benign neoplasm. Lipomas are the commonest benign tumours in the spermatic cord.

175
Q

A 62-year-old man presents with bilateral testicular enlargement. Ultrasound reveals bilateral
smoothly enlarged testes with diffuse hypoechoic areas and normal epididymis. Which of the
following is the most likely diagnosis? [B2 Q42]

a. Lymphoma
b. Metastasis from prostatic cancer
c. Seminoma
d. Tuberculosis
e. Leydig cell tumour

A

Lymphoma

This is the wrong age group for Leydig cell tumour (childhood) and seminoma (around 40
years). Lymphoma is the commonest tumour in this age group. It is more likely to have a
diffuse hypoechoic appearance and be bilateral as compared to metastasis, which usually
presents with multiple focal lesions. Tuberculosis of the testis is most often secondary to
epididymitis.

176
Q

Non seminomatous Germ Cell Tumour is diagnosed in a 30-year-old man. Which is the single
best answer? [B3 Q17]

A. Non seminomatous Gem Cell Tumour form the majority of testicular Gem Cell Tumour
B. Left-sided tumours frequently metastasize to left PA nodes just above level of left renal vein
C. Right-sided tumours spread initially to aortocaval lymph nodes
D. Echelon nodes are usually left-sided nodes lateral to PA nodes
E. Imaging appearances of nodal disease at late relapse is often more predictable than at initial
presentation

A

Right sided tumours spread initially to aorto-caval lymph nodes

Late relapse > 2 years after CR, in absence of second primary tumour. Occurs in up to 7% of
patients with less predictable pattern of nodal spread.

NSGT form 40% of testicular GCT compared with 60% seminomatous.
Left-sided tumours metastasize to left PA nodes below the level of the renal vein.
Echelon nodes are usually right-sided, anterior to right psoas

177
Q

A 40-year-old man has a testicular ultrasound scan, which demonstrates a multilobular mass
that is homogeneous and hypoechoic with Doppler flow seen in internal hypoechoic bands.
Which of the following is the most likely diagnosis? [B4 Q7]

a. teratoma
b. lymphoma
c. metastasis
d. seminoma
e. focal infarction

A

Seminomas

Ninety-five per cent of testicular tumours are germ cell tumours. Others include sex-cord and
stromal tumours such as Leydig cell and Sertoli cell tumours. Primary lymphoma and
metastases can also occur in the testicle. Non-seminomatous germ cell tumours include
teratoma, embryonal carcinoma and choriocarcinoma, but these affect a younger population
of 20–30 years. Ultrasound scan is the investigation of choice for detection of a testicular
tumour and for assessing normality of the contralateral testicle. Seminomas present with
mass or pain and are generally lobulated masses on ultrasound scan with hypoechoic
fibrovascular septations in which colour flow can be visible. T2W MRI demonstrates uniform
intermediate signal with band-like low-signal septa. There is contrast enhancement,
especially of the septations. They rarely calcify, but, if they do, the calcification is speckled or
stippled

178
Q

A 20-year-old man has radical orchidectomy for a non-seminomatous germ cell tumour. A CT
of the thorax, abdomen, and pelvis shortly after surgery shows no lymphadenopathy or
metastasis. Which of the following is the most appropriate follow-up regimen? [B4 Q19]

a. repeat CT of the chest, abdomen, and pelvis only in response to symptoms suggesting
recurrence
b. repeat CT of the chest, abdomen, and pelvis only when serum tumour markers rise
c. serial serum tumour marker measurement with yearly CT of the chest, abdomen, and pelvis
d. serial serum tumour marker measurement with 3-monthly CT of the chest, abdomen, and
pelvis for 1 year followed by 6-monthly CT of the chest, abdomen, and pelvis for 1 year
e. 3-monthly whole-body PET/CT

A

Serial serum tumour marker measurement with 3-monthly CT of the chest, abdomen, and
pelvis for 1 year followed by 6-monthly CT of the chest, abdomen and pelvis for 1 yea
r.

Stage 1, non-seminomatous, germ cell tumour patients should enter a surveillance
programme of this type following orchidectomy. Such programmes are rarely used for
seminoma, particularly when retroperitoneal radiation treatment is used. Rising tumour
markers between surveillance scans or thereafter should provoke CT of the chest, abdomen,
and pelvis plus ultrasound scan of the remaining testicle. If no new disease is identified, MRI
of the brain is indicated

179
Q

On ultrasound scan, a 30-year-old man is found to have bilateral testicular microlithiasis and
unilateral testicular atrophy. There is a history of orchidopexy of the atrophic testicle. Which
of the following is the most appropriate management? [B4 Q20]

a. discharge
b. self-examination only
c. follow-up clinical examination and surveillance sonography
d. further investigation with MRI
e. testicular biopsy

A

Follow-up clinical examination and surveillance sonography

Atrophic mal-descended testes are at higher risk of developing malignancy, particularly
seminoma, even after orchidopexy. The increased risk applies to the contralateral testicle also.
Microlithiasis is also associated with testicular cancer. In combination, these features require
clinical and sonographic follow-up.

180
Q

A 40-year-old man has a right-sided intratesticular mass of indeterminate ultrasound
appearance. A CT scan of the chest, abdomen and pelvis reveals an enlarged, round, cystic,
right para-caval lymph node just caudal to the right renal vein. Other enlarged cystic lymph
nodes are demonstrated in the mediastinum. No enlarged suprarenal or retro-crural nodes are
seen. Considering these findings, which tumour type is most likely to be found in the
orchidectomy specimen? [B4 Q51]

a. epidermoid tumour
b. malignant teratoma
c. seminoma
d. metastatic lung squamous cell carcinoma
e. adenomatoid tumour

A

Malignant teratoma

Seminoma tends to spread to contiguous nodes with the primary sites being, for a right-sided
testicular tumour, right para-caval and inter-aortocaval nodes just below the junction of the
right renal vein and inferior vena cava. Left-sided testicular tumours usually spread first to
nodes just caudal to the left renal vein. Teratoma and other malignant, non-seminomatous,
germ cell tumours of the testes can occupy mediastinal nodes without such direct cranial
extension along the nodal groups.

Nodes involved by seminoma tend to be soft-tissue density while nodes inhabited by non-
seminomatous, germ cell tumours are frequently cystic.

Metastases to the testicle are rare on account of the testicle being an immunologically
privileged site with a blood–testicular barrier.

Adenomatoid tumour is a benign epididymal lesion.

Epidermoid tumour is also benign, appearing as an intratesticular, hypoechoic lesion,
characteristically with an echogenic capsule. Classically, it assumes an ‘onion-skin’
appearance of concentric echogenic layers. Internal shadowing can be produced by
calcification.

181
Q

A 20-year-old man presents with a 4-week history of a scrotal mass. There has been no trauma
and no pain. Ultrasound scan confirms an intratesticular, partly cystic, heterogeneous mass.
Which of the following tumour markers is most likely to be elevated? [B4 Q82]

a. CA-125 and AFP
b. CA-15-3 and CEA
c. PSA
d. CA-19-9
e. AFP and b-hCG

A

AFP and b-hCG

The patient’s age and ultrasound findings favour a non-seminomatous germ cell tumour.
These are associated with elevated serum AFP and b-hCG. Together, CA-125 and AFP are
associated with hepatocellular carcinoma, while CA-125 is also associated with ovarian cancer.
CA-15-3 and CEA together are serum markers associated with breast cancer. PSA is associated
with prostate cancer as well as several benign prostate conditions. CA-19-9 is mainly
associated with malignancy of the pancreas and biliary tree

182
Q

A 27-year-old man was diagnosed with testicular seminoma, not invading the scrotal sac.
Which of the following lymph node groups is most likely to be involved? [B5 Q50]

(a) Ipsilateral inguinal nodes
(b) Para-aortic nodes
(c) Common iliac nodes
(d) Supraclavicular nodes
(e) Retro-crural lymph nodes

A

Para-aortic lymph nodes

The lymphatics from the testis accompany the veins to the retroperitoneal nodes between
the bifurcation and the kidneys. The local inguinal nodes are involved only if there is invasion
of the scrotal wall.

183
Q

A 24-year-old man is referred for an ultrasound examination following blunt trauma to the
scrotum. Which of the following is not true? [B1 Q41]

a. The left testis is more susceptible to blunt trauma
b. Intratesticular haematomas need to be followed up until resolution
c. Penetrating injuries are more likely to be bilateral compared to blunt injuries
d. An ultrasound finding of an intact tunica albuginea allows the confident exclusion of a
testicular rupture in the absence of a haematocele
e. An atrophic testis is more likely to dislocate

A

The left testis is more susceptible to blunt trauma

The testis suffers blunt trauma against the thigh or the symphysis pubis and the right testis,
being higher, is more susceptible. Intratesticular haematomas should be followed up to
resolution to rule out an underlying neoplasm and rule out any ensuing complications such
as abscess formation which may necessitate orchidectomy.

184
Q

A 14-year-old boy slides off his saddle while cycling, injuring his testicles. On ultrasound scan,
one testicle has a para-testicular complex cystic mass, loss of testicular outline and several
avascular planes within the testicle. Which of the following is the most appropriate
management? [B4 Q44]

a. discharge with analgesia
b. admission for analgesia
c. admission for elective surgery
d. admission for surgery the following day
e. immediate surgical intervention

A

Immediate surgical intervention

Testicular rupture is described; it is an indication for immediate surgical intervention to
salvage the testicle and prevent anti-sperm antibody development (testicles are immune-
privileged sites). Other consequences of testicular trauma are fracture, haematoma, and
haematocele. Associated torsion may occur, due to trauma-stimulated, forceful, cremasteric
muscle contraction.

185
Q
A
186
Q

A portal venous phase abdominal CT scan of a 65-year-old man demonstrates an ill-defined,
rounded area 4 cm in diameter within a kidney. It is heterogeneous but predominantly of
attenuation value above 70 HU. It contains small dense calcific foci. Which additional feature
suggests that the lesion is more likely to be a renal cell carcinoma than a transitional cell
carcinoma of the collecting system? [B4 Q95]

a. thickened indurated pelvicalyceal wall
b. central location of the tumour with centrifugal expansion that compresses renal sinus fat
c. renal parenchymal invasion with renal contour preservation
d. renal vein thrombus
e. further mass arising from the urinary bladder wall

A

Renal Vein Thrombus

Differentiation of renal cell from transitional cell carcinoma is helpful for planning surgical
treatment since transitional cell carcinoma of the renal collecting system requires the more
extensive surgical procedure of nephroureterectomy. Renal vein thrombus is seen with renal
cell carcinoma while all other options given are features of transitional cell carcinoma of the
kidney. Delayed contrast CT offers a pyelographic phase on which collecting system, ureter
and bladder filling defects are clearly demonstrated. A urothelial field effect can occur,
resulting in multiple transitional cell carcinomas throughout the renal tract. Renal cell
carcinoma, as it expands, tends to distort the renal outline and is more likely to be peripheral
and exophytic